You are on page 1of 55

BREAK UP OF QUESTIONS

SUBJECT Indian History


No. of Questions
UNIT Topic
Given
Salient features of Indus Valley Civilization and Vedic age 4
Emergence of Buddhism and Jainism 1
1. Mauryan Empire: Their administration, Socio-Economic and Religious Conditions, Art
2
Ancient and Architecture, Literature
History Gupta Empire: Their administration, Socio-Economic and Religious Conditions, Art and
2
Architecture, Literature
Harshavardhana and his Achievements 1
Sub Total 10
The Chola Administrative System 0
Delhi Sultanate: Their Administration, Socio-Economic and Religious Conditions,
3
Art and Architecture, Language and Literature
2.
The Mughal Empire: Their Administration, Socio-Economic and Religious Conditions,
Medival 2
Art and Architecture, Language and Literature
History
Bhakti and Sufi Movements 5
Shivaji and the rise of Maratha Empire 0
Advent of Europeans
Sub Total 10
1857 Revolt and its Impact 1
Rise and Consolidation of British Power in India - Changes in Administration, Social and
4
3. Cultural Spheres
Modern Social and Religious Reform Movements in the 19th and 20th Century 1
History Indian National Movement: it's various stages and important contributors and
4
contributions from different parts of the country
Post Independence Consolidation and Reorganization within the country
Sub Total 10
TOTAL 30

AMIGOS IAS 1
SUBJECT Geography
Interior of the Earth - Major Landforms and their features 3
Soils and Vegetation 1
Drainage System India 1
1.
Climate: Structure and Composition of Atmosphere 2
Major Physiographic Features of India 1
Major Physiographic Features of AP 2
Sub Total 10
Natural resources and their distribution 2
Agriculture and Agro-based Activities 3
Distribution of Major Industries 2
2.
Major Industrial Regions 1
Major Industrial Regions of AP 1
Natural resources and their distribution of AP 1
Sub Total 10
Demographics 3
Urbanization and Migration 2
3. Human Development 3
Racial groups 1
Tribal groups 1
Sub Total 10
TOTAL 30

AMIGOS IAS 2
SUBJECT Indian Society
Family 1
Marriage 3
Kinship 1
Sturcutre of
Caste 1
Indian
Tribe 1
Society
Ethnicity 1
Religion 1
Women 2
Sub Total 11
Casteism 1
Communalism 1
Regionalisation 1
Social Issues
Crime against Women 0
Child Abuse and Child Labour 4
Youth Unrest and Agitation 1
Sub Total 8

Public Policies and Welfare Programmes, Constitutional and Statutory


5
Welfare Provisions for Schedule Castes, ScheduleTribes, Minorities, BCs
Mechanism Public Policies and Welfare Programmes, Constitutional and Statutory
6
Provisions for Women, Disabled and Children.
Sub Total 11
TOTAL 30

AMIGOS IAS 3
SUBJECT Current Affairs
Polity and Governance 1

Economy 0

Geography, Environment, places in news 3

History and culture 1


Regional
Programmes and Schemes 2

Personalities in news 1

Awards and Reports 2

Sports 0
Sub Total 10
Polity and Governance 2

Economy 1

Geography, Environment, places in news 2

History and culture 0

Programmes and Schemes 1


National
Personalities in news 0

Sports 1

Science and Tech 2

Awards and Reports 1

Miscellaneous 0
Sub Toal 10
India and Neighborhood 1
India and Extended Neighborhood 0
India and Major Powers-USA, Russia, China, Etc. 1
India and Groupings 1
Rigional Groupings 1
Inter-national International Organisations 1
Miscellaneous Topics 2
Indices and Reports 1
Places in News 2
Exercises and Operations 0
Sub-Total 10
TOTAL 30

AMIGOS IAS 4
SUBJECT Mental Ability
Data Interpretation (Bar Graph) 4
Series 3
Calendar 1
Blood Relation 1
Profit and Loss 1
Number System 3
Percentage 2
Ratio and Proportion 1
Averages 1
Time & Work 2
Time & Speed 2
Data Interpretation (Pie chart) 5
Statement & Course of action 2
Statement & Assumption 2
Total 30
GRAND TOTAL 150

AMIGOS IAS 5
APPSC – GROUP-II, Prelims Test (14-02-2024)
1. Match List-I(Administrative tiers of Mauryan are seen as the greatest artistic creations
State) with List II(Heads) and select the of the Harappan culture.
correct answer using the codes given below 1. A seal is a stamp inscribed with the
the lists: symbol/name of its owner, while its
List I List II impression on any material is called a
1. Province A. Gopa sealing. Hence, statement 1 is correct.
2. District B. Gramika 2. Both seals and sealings were essentially
3. Village C. Aryaputra marks of ownership of the sender meant to
4. A group of Villages D. Sthanika guarantee the quality of a product being
sent to faraway lands.
Codes:
a. Goods were sent from one place to
1) 1-C, 2-D, 3-B, 4-A
another, possibly in a bag, its mouth tied
2) 1-D, 2-C, 3-A, 4-B
with a rope. Its knot was sealed using a
3) 1-B, 2-A, 3-C, 4-D sealing of wet clay and one or more seals
4) 1-A, 2-B, 3-D, 4-C were pressed onto it. An intact sealing
Answer: 1 meant that the bag had not been tampered
Explanation: with.
Administrative Structure of the Mauryan 3. More than 2000 seals have been
Empire: discovered; they are mostly made of soft
1. Centre-headed by swami/raja river stone, steatite and are generally
2. Province-headed by rashtriya/ square in shape (2 by 2 dimension), though
kumara/aryaputra some round seals have also been found.
3. District headed by sthanika 4. Some were also made of copper, gold,
4. Sthaniya (800 villages) ivory, and terracotta. Hence, statement 2 is
incorrect. If you eliminate this option,
5. Dronamukha (400 villages)
you’ll get the answer.
6. Karvatika (200 villages)
5. They were mainly used for commercial
7. Samgrahana (10 villages)-headed by gopa
purposes; but sometimes also possibly
8. Village headed by gramik used as tokens for buying and selling, or as
2. Consider the following statements about identification markers or even as amulets.
Harappan Seals: 6. The designs on the seals include a variety
1) It is a stamp inscribed with the name of its of relief figures/intaglio motifs (human,
owner animal, composite and geometric figures)
2) They are made of only steatite along with inscription in a semi-
3) The rhinoceros is the most frequently pictographic script which probably
depicted animal on the Harappan seals contained the name of the owner.
Which of the statements given above are a. The script has not yet been deciphered.
correct ? 7. The animal motifs used include the bull
1) 1 and 2 2) 1 and 3 (Zebu), rhinoceros, tiger, elephant, Indian
3) 2 and 3 4) 1,2 and 3 bison, snake, antelope, and gharial, which
Answer: 2 may also have cultic significance. The
Explanation: rhinoceros is the most frequently depicted
animal on the Harappan seals.Hence,
Seals and Sealings:
statement 3 is correct.
 Seals are possibly the most distinctive
artefact of the Harappan civilization, and

AMIGOS IAS 1
NOTE: The source of above question is from Answer: 1
page no. 63, class 11th NCERT. The most Explanation:
frequently depicted animal in seals is unicorn Numismatics
and not humped bull. Here (Page No. 63) 1. The study of coins is called numismatics.
NCERT is mentioning unicorn as rhinoceros.
2. Largest number of punch-marked coins
Hence statement 3 is correct.
found under the Mauryas
3. Largest number of coins in general found in
3. Which of the following is the largest among the Post-Mauryan period)
all the excavated Harappan sites ?
4. Largest number of clay coins found under
1) Mohenjodaro 2) Harappa the Kushanas
3) Dholavira 4) Rakhigarhi 5. Largest number of potin and lead coins
Answer: 4 found under the Satavahanas
Explanation: 6. Largest number of gold coins found under
 The harappan sites varied immensely in the Guptas
size and function- some were large cities 5. Which of the following Inscriptions talks
while others were mere pastoral camps : about the enlargement of the stupa of
1. Largest Sites: Buddha Kanakamuni to double its size ?
a. Rakhigarhi (350 ha, after recent 1) Rummindei Pillar Inscription
excavations), 2) Sarnath Pillar Inscription
b. Mohenjodaro (200 ha) 3) Sanchi Pillar Inscription
c. Harappa (150 ha) 4) Nigalisagar Pillar Inscription
d. Dholavira (100 ha) and Answer: 4
e. Lurewala and Ganweriwala in Cholistan (a Explanation:
desert area in Pakistan that forms part of  Rummindei/Lumbini/Paderia Pillar
the Greater Thar Desert). Inscription (in Nepal)
2. About 50 ha 1. It is commemorative in character and
a. Nagoor states that king Devanampiya Piyadasi,
b. Tharo Waro Daro in Sindh. when he had been consecrated 20 years,
3. 10-50 ha came in person and worshipped here.
a. Judeirjodaro 2. Because here Sakyamuni Buddha was born.
b. Kalibangan. 3. The king caused a huge stone wall to be
4. 5-10 ha made, a stone pillar to be erected, Lumbini
a. Amri village was freed from religious cesses and
b. Lothal made to contribute one-eighth share (only,
as land revenue).
c. Chanhudaro
Nigliva/Nigalisagar Pillar Inscription (in Nepal)
d. Rojdi
1. It states that when the king Devanampiya
5. 1-5 ha
Piyadasi had been consecrated 14 years, he
a. Allahdino
enlarged the stupa of Buddha Konakamana
b. Kot Diji (Kanakamuni, a mythical Buddha) to
c. Rupar double its size.
d. Balakot 2. And when he had been consecrated 20
e. Surkotada. years, he came in person, worshipped
4. The Largest number of punch-marked coins here, and had a stone pillar erected.
were found under the 6. Consider the following:
1) Mauryas 2) Kushans
3) Satavahanas 4) Guptas

AMIGOS IAS 2
“The father gifts his daughter after saying to Codes:
the couple, 'May both of you perform your 1) 1-D, 2-B, 3-A, 4-C
religious duties together’.” 2) 1-A, 2-C, 3-D, 4-B
Which type of marriage is described above ? 3) 1-C, 2-A, 3-B, 4-D
1) Prajapatya 2) Rakshasa 4) 1-B, 2-D, 3-C, 4-A
3) Brahma 4) Paishacha Answer: 3
Answer: 1 Explanation:
Explanation: Important Mandals of Rig veda:
Types of Marriages 1. Mandal 3 : Gayatri Mantra
● Brahma 2. Mandal 6 : Mention of term Hariyupiya
○ One in which the father adorns his 3. Mandal 7 : Battle of 10 kings
daughter with costly clothes and 4. Mandal 9 : Dedicated to Lord Soma
ornaments, and gifts hers a man learned in
5. Mandal 10:
the Veda.
a. Nadistuti Sukta(Hymn dedicated to rivers)
● Daiva
b. Nasadiya Sukta( dealing with speculations
○ The father gives his daughter in marriage to
related to the creation of universe)
an officiating priest, during the course of a
c. Purusha Sukta(dealing with division into 4
sacrifice
varnas)
● Arsha
8. With respect to Harshavardhana, Consider
○ The father gifts his daughter, after taking a
the following statements :
pair of cattle as a custom and not as sale of
1. He followed the policy of religious
daughter
tolerance
● Prajapatya
2. He held Prayag assembly in honour of
○ The father gifts his daughter after saying to
Hiuen-Tsang
the couple, 'May both of you perform your
3. He held Kannauj assembly to freely donate
religious duties together’.
his accumulated wealth to the people.
● Asura
Which of the statements given above are
○ The father gives the daughter to the groom
correct ?
in exchange for wealth.
1) 1 only 2) 2 and 3
● Gandharva
3) 1 and 3 4) All of the above
○ It is love marriage through mutual consent.
Answer: 1
● Rakshasa
Explanation:
○ One in which the girl is forcibly abducted,
Kannauj and Prayag Religious Assemblies (643
after beating or killing her relatives.
CE)
● Paishacha
 Harshavardhana was a Shaivite, but was
○ When a man has sex with a girl while she is
tolerant towards other religions and
asleep, unconscious, or mentally
extended his support as well. In the later
disordered.
part of his life, Harshavardhana became a
7. Match List-I (Rigvedic Mandal) with List II devotee and patron of Mahayana
(Subject matter) and select the correct Buddhism. Hence,statement 1 is correct
answer using the codes given below the lists:
 In the year 643 CE, Harshavardhana
List I List II
summoned two religious assemblies.
1. Mandala 3 A. Purusha Sukta
1. One was a great Buddhist assembly at
2. Mandala 10 B. Battle of 10 Kings Kannauj.
3. Mandala 7 C. Gayatri Mantra
4. Mandala 9 D. Lord soma

AMIGOS IAS 3
a. The Kannauj assembly was held in honour Criticism
of Hiuen-Tsang, for whom he had great ● decline of settlements outside Indus not
affection and regard. explained by this theory
b. In this Buddhist assembly, Harshavardhana ● a river cannot be dammed by tectonic
donated liberally to Buddhist monasteries. effects
c. Hence,statement 2 is incorrect Shifting of Indus (destroyed Mahenjodaro):
2. Kannauj assembly was followed by Prayag (LAMBRICK)
assembly, in which Harshavardhana Evidence
conducted the charitable ceremony. ● silt is there which is sandy
a. The Prayag assembly was called Moksha ● sand silt is not due to floods
Parishad, which was to be held at the end Criticism
of every five years.
● This only explains desertion of
b. Hence, statement 3 is incorrect Mohenjodaro but not its decline
3. According to Banabhatta and Hiuen-Tsang, Increasing Aridity: (AGRAWAL & SOOD)
Harshavardhana had the reputation for his
Evidence
charitable works.
● 2nd millenium BC a period of increased arid
4. At the end of every five years,
conditions.
Harshavardhana in the Prayag assembly
● decline of agriculture
would freely donate his accumulated
wealth to the people. ● drying up of Ghaggar
5. Hiuen-Tsang has given the account of the Criticism
Prayag assembly of 643 CE. ● Why Ghagghar dried up, not worked out
a. He wrote that Siladitya (Harsha) performed yet.
the ceremony of 'dana' (donation), which Aryan Invasion: (Wheeler)
lasted for about three months. Evidence
b. He donated his clothing, his jewellery, etc. ● Human skeletons lying on the streets
c. His generosity was indeed unbounded and ● Rigveda refers to fortress which god
unparalleled. Purandara, destroyed
9. The Ecological Imbalances theory to explain ● The geographical area of Rigveda Aryans
the decline of Indus Valley civilisation was include Punjab-Ghaggar region
proposed by ● Vedas mention a place called Hariyupiya on
1) Lambrick 2) Agarwal and Sood Ravi river where Aryans fought a battle. It
3) Mortimer Wheeler 4) Fairservis has been identified with Harappa
Answer: 4 Criticism
Explanation: ● Difference between 1800-1500 BC (300
Theories of Decline:- years)
Flood and Earthquake: (RAIKES) Ecological Imbalance: (Fairservis)
Flood ● population increase, city grew and land
veduced, forest decreased
● Silt deposits above ground level
● soil exhaustion of area
● Houses built on silt covered debris
● with forests disappearing there were more
Earthquake
floods and droughts
● Indus area is an earthquake-prone zone
● townsmen moved to Gujarat region and
● The earthquake raised the level of the sea,
Eastern area
causing floods in cities flood plains,
● this decline completed by raids and attack
blocking passage. of rivers water
by nearby settlements
● earthquake caused shifting of land away
from sea-coast affecting commercialities

AMIGOS IAS 4
10. Nitisara, a book on statecraft addressed to including four Maths on the line of
the king was written by Buddhist monasteries. These efforts of
1) Kalidasa 2) Vishnu Sharma Shankaracharya systematised the Hindu
3) Amoghavarsha 4) Kamandaka religion which was on the verge of
Answer: 4 degeneration with the onslaught of Islam
and growth of Buddhism.
Explanation:
● He redefined the Vedantic philosophy by
Nitisara by Kamandaka
stressing its doctrine of Monism or Advaita.
 Nitisara by Kamandaka (4th century CE) is
○ Through Advaita doctrine, he proved that
a work on statecraft addressed to the king.
the supreme reality is Brahma and the rest
Panchatantra by Vishnusharman- is Maya or Illusion.
1. Though attributed to sage Vishnusharman, ○ There is no dualism i.e..,the world and the
the date and authorship of this work is individual soul is one and that is Brahma.
considered uncertain.
○ According to him, Brahma is formless and
2. It is an example of nidarshana-meaning a without any quality.
work that explains what should or should
○ One can realise this formless Brahma
not be done through illustration.
through meditation and the right
3. The work was possibly composed in the knowledge as given in Upanishads.
Vakataka empire when the sage needed to
○ According to him the way to salvation is to
instruct three princes in the subject of niti
realise this formless Brahama.
(policy) through engaging stories.
● Through his debate on monism or Advaita
4. Panchatantra stories are written in prose,
doctrine, Shankaracharya established the
interspersed with verses.
supremacy of Vedanta philosophy against
5. In most stories, animals play an important the Sankhya philosophy of Mahayana
role. Buddhism. The Vedantic Philosophy of
11. Match List-I(Philosopher) with List Shankaracharya was revived by
II(Doctrine) and select the correct answer Vivekananda in the second half of the
using the codes given below the lists: nineteenth century.
List I List II Ramanujacharya and Vishishtadvaita Doctrine
1. Shankaracharya A. Vishishtadvaita ● The twelfth-century Hindu philosopher
Doctrine Ramanujacharya from Tamil Nadu reacted
2. Ramanujacharya B. Dvaita Doctrine against the Shankara's Advaita doctrine
3. Madhavacharya C. Dvaita-Advaita and its concept of Nirgunabrahma or God
Doctrine without attributes.
4. Nimbarkacharya D. Advaita Doctrine ● Ramanujacharya preached the doctrine of
Codes: Vishishtadvaita or qualified monism.
1) 1-C, 2-B, 3-A, 4-D 2) 1-B, 2-C, 3-D, 4-A ● According to this doctrine, there is an
3) 1-D, 2-A, 3-B, 4-C 4) 1-A, 2-D, 3-C, 4-B individual soul or Atma which is one with
the Ultimate reality or Brahma.
Answer: 3
○ The Brahma is Saguna Brahma having form
Explanation :
and attributes.
Shankaracharya and Advaita Doctrine
○ The physical world, soul, and God are real
● In the ninth century, Hindu revivalist saint and not illusory as claimed by
Sankaracharya from Kaladi in Kerala gave a Shankaracharya.
philosophical impetus to the emerging
● Ramanujacharya emphasised that the
Bhakti cult. He reinterpreted ancient Hindu
grace of God obtained through Bhakti was
scriptures and established the Hindu
more important than the knowledge about
centres of learning all across the country
God to attain salvation.

AMIGOS IAS 5
○ He enrolled himself into the Vaishnava 2. His lingayat philosophy is close to the
order and followed Nayanaras Bhakti Vishishtadvaita doctrine of Ramanuja
Marga. 3. He referred to Mahadevi as ‘Akka'
○ For him, worship of Vishnu was a way to 4. Prime Minister Narendra Modi unveiled his
attain salvation. statue in London on the bank of the River
● Ramanujacharya's doctrine of Thames
Vishishtadvaita popularised the Which of the statements given above are
Vaishnavite Bhakti tradition in eastern correct ?
India. 1) 1,2 and 3
Madhavacharya and Dvaita Doctrine 2) 2,3 and 4
● In the thirteenth century, Madhavacharya 3) 1,3 and 4
from Karnataka propagated the Doctrine of 4) All of the above
dualism which was in contrast with the
Answer: 4
Advaita Vedanta of shankaracharya and
Explanation:
Vishishadvaita of Ramanuja.
● According to Adita Vedanta, the World is  Basaveshwara or Basavanna and Lingayat
not an illusion and there is a difference Cult
between Atman or self soul and Brahma or  Basaveshwara or Basavanna was a 12-
ultimate reality. century Kannada Shaivite saint-poet and a
Nimbarkacharya and Dvaita-Advaita Doctrine leading social reformer of this time. He
lived during the rule of Kalyani Chalukyas
● He was the contemporary of
and Kalachuri rulers. He rose to
Ramanujacharya and Madahvacharya.
prominence under King Bijjala II of
● Nimbarka was a Vaishnav philosopher and
Kalachuri under whom he served as a
commentator.
Prime Minister.According to tradition,
● He is known for propagating the Vaishnava Basavanna was the founder of the
doctrine of Bhedabheda Dvaitadvaita i.e. Lingayat cult-one of the Shiva sect.
doctrine of duality and non-duality. However, it is evident from research that
● Nimbarakacharya's doctrine of Basavanna was inspired by the already
Dvaitadvaita talks of duality and existing Shiva cult known as Veerashivas,
nonduality. known for ardent and heroic Shiva
○ He defines three categories of existence, worshippers. He revived and organised
God,soul, and matter. the tradition of Veerashaivas into the
○ God (Isvara) is an absolute reality and Lingayat cult. Basava's Lingayat
exists independently. philosophy is close to the Vishishtadvaita
○ Though both soul and matter exist doctrine of Ramanuja. In Lingayat, the
independently, they are dependent upon individual soul or Atma is the body of God,
God. and that there is no difference between
○ For him, Krisha is an absolute reality, and Shiva and Atman.The Lingayats cult
Radha-his consort and nature. attracted large people from lower castes,
○ He, therefore, propagated Krishna-Radha men and women. The cult rejected temple
Bhakti. worship and ritualistic worship led by
Brahmins. It introduced direct and
● Nibarakacharya's order is known as the
personalised worship of Lord Shiva. It
Nimbarka Sampradaya which still exists.
opposed any image worship of Shiva
12. With respect to Basaveshwara, consider the
except other than Linga worship.
following statements:
Basavanna was a radical social reformer of
1. He served as a Prime Minister under King Karnataka. He challenged the dominance
Bijjala II of Kalachuri dynasty of Brahmanical religion and opposed any

AMIGOS IAS 6
discrimination based on caste and gender.  Hence, all the statements are correct.
He opposed many Brahmin practices such 13. What are the reasons for relocation of the
as temple worship, wearing sacred capital of Delhi sultanate from Delhi to
threads, etc. Instead of scared-thread, he Daulatabad ?
introduced the ritual of wearing Ishtalinga 1. Central location
necklace, with an image of the Shiva Linga 2. A base for the administration and
which was worn by every Lingayat consolidation of South India
regardless of his or her birth. Under his
3. Mongol Menace
leadership, a people's movement known
as the Sharana movement emerged. The 4. Promoting Muslim culture in the south
movement opposed social injustice and Select the correct answer using the codes
social oppression carried by the caste given below :
system and gender inequality. To 1) 1,2 and 3 2) 1,2 and 4
challenge caste order, Sharanas organised 3) 1,3 and 4 4) All of the above
inter-caste weddings. Answer: 4
 Basavanna set up a spiritual academy Explanation:
called Anubhava Mantapa, in which  The reasons attributed by historians for
people from all castes and communities the relocation of the capital to Daultabad
were welcomed to engage in spiritual and are:
social discourses. 1. Daultabad was centrally located to the
 The Lingayat literature is compiled in empire and more strategically placed as
Shunya Sampadane, an anthology of compared to Delhi.
Kannada poems which includes the 2. Daulatabad served as a base for the
Vachanas and dialogues of several administration and consolidation of South
Lingayat saints.The government of India.
Karnataka has honoured this great 3. Unlike Delhi, Daultabad was very much
reformer by issuing a stamp on his name. safe from the foreign invasions, such as
Recently, the government of India under Mongols.
Prime Minister Nar- endra Modi unveiled
4. Daultabad would serve as the centre for
his statue in London on the bank of the
promoting Muslim culture in the south and
River Thames.
had access to the wealth of the South.
 Mahadevi Akka or Akka Mahadevi Hence, all the statements are correct.
 One of the prominent female saint poets 14. Consider the following statements about the
associated with the Lingayat cult was introduction of Token Currency by
Mahadevi Akka or Akka Mahadevi (c. Muhammad Bin Tughluq :
1130-1160). A fervent devotee of Shiva,
1. Global Shortage of silver was the main
she became ascetic in her early life and
reason for the introduction of it.
joined the Sharanas.Akka Mahadevi
considered Shiva as her husband and 2. He introduced copper and brass coins
expressed her spiritual relationship with equivalent to the value of silver coins.
her Lord in Vachana poems. Bassavanna 3. As a result of it the gold and silver coins
and other Sharna members referred to disappeared from the market.
her ‘Akka' meaning elder sister. She was Which of the statements given above is/are
among the earliest women reformers who incorrect ?
worked for women emancipation. 1) 1 and 2 2) 2 and 3
Together, Basavanna and Mahadevi Akka 3) 1,2 and 3 4) None
carried the message of Virashaivism or Answer: 4
heroic Shaivism in Karnataka through their Explanation:
Vachanas.

AMIGOS IAS 7
 Introduction of Token Currency (1329- repercussions. This created
1330 CE) miscommunication amongst people who
 Another bold experiment of Muhammad instead of cooperating with the Sultan
which failed miserably was the introduc- manipulated the opportunity.
tion of token currency. In 1329-30 CE, the 15. Who among the following Bhakti saints
Sultan introduced copper and brass coins expressed Krishna Bhakti through ecstatic
equivalent to the value of silver coins. songs and dance ?
Muhammad issued the token currency 1) Kabir Das
experiment after witnessing the success 2) Ramananda
of the paper money project of China. 3) Chaitanya Mahaprabhu
Historians are of the opinion that there 4) Lal Ded
was worldwide shortage of silver at that
Answer: 3
time and India too faced its serious
shortage. Therefore, the Sultan was Explanation:
forced to issue token currency. Hence,  Chaitanya Mahaprabhu (1486-1533 CE)
statement 1 is correct.  Chaitanya Mahaprabhu was a renowned
 In the new scheme, the value of the Vaishnava saint poet from Bengal. He was
copper coin or jital (paisa) was kept at par born into a Brahmin family as Viswambhar
with the silver coin or tanka (rupee). So Mishra in 1486 CE in Kolkata. After he
now there was silver tanka, and also studied the scriptures, he married and
copper tanka. Hence,statement 2 is started a normal life. However, after
correct. meeting his guru Iswar Puri at Gaya, he
renounced material life and became
 This scheme of token currency resulted in
ascetic and was called upon as Chaitanya
a blunder. People began to forge coins in
Mahaprabhu.Chaitanya Mahaprabhu
large numbers. They minted token coins in
expressed his Krishna Bhakti through
their houses. The counterfeit copper coins
ecstatic songs and dance. His Kirtanas or
replaced the gold and silver coins
devotional songs highlight Raasleela or
completely. People used fake token
currency for payment of revenue and the celestial dance play of Krishna and
taxes and hoarded gold and silver coins at Radha. Chaitanya Mahaprabhu through
their homes. The result was that the gold Radha Krishna Bhakti propagated the
Krishna Consciousness movement and the
and silver coins disappeared from the
chanting of the 'Hare Krishna' mantra.
market. Hence, statement 3 is correct.
This adversely affected the trade and 16. In the context of Mughal Administration, the
commerce particularly the foreign trade office of Muhatribs is related to
as foreign merchants suspended their 1) Revenue collector
business transactions with 2) Censors of public morals
India.Historians have blamed the Sultan 3) Local Record keeper 4) Spies
for the failure of the scheme. According to Answer: 2
them, the Sultan failed to take necessary Explanation
measures to check the imitation of coins.  Aurangzeb took the following puritanical
After realising the failure of his scheme, measures to convince and gain popularity
the Sultan withdrew the entire token among the Sunni Muslims who formed the
currency. The people were asked to return
majority among the Muslims:
the token coins and, in exchange, were
1. Appointments of Muhatribs or public
paid back gold and silver coins by the royal
censors of public morals to ensure that the
treasury. Historians felt that the Sultan's
people acted according to the teaching of
scheme of token currency was a futuristic
the Quran. Muhatribs were appointed in
idea. He was not able to foresee the

AMIGOS IAS 8
every important town and city to curb the  Fatwa-i-Alamgiri was a compilation of
practices of drinking, gambling and Muslim laws. It deals with statecraft,
prostitution. They had the power to punish ethics, military strategy, economic policy,
those who were guilty of heresy or justice and punishment. The book was
blasphemy. written by Aurangzeb along with other
2. Regular payments to Imams and Muzzims. Muslim scholars. This Muslim digest
3. Order for the repair of mosques and other became a rule book on law and principle
Muslim religious places. during Aurangzeb's reign.
4. Prohibition for inscribing Kalima or Muslim 18. Which of the following are the observations
prayer on the coins. This practice was made by Alberuni on Indian society in his text
considered un-Islamic. Kitab al Hind ?
5. Banned alcohol and other intoxicating 1. Closed society
drugs. In addition, the cultivation of bhang 2. Stagnant knowledge
in the empire was prohibited. 3. Social evils
6. Strong measures were taken against Sufi Select the correct answer using the code given
saints particularly those who were below:
supporters of Dara Shikoh. 1) 1 and 2 2) 1 and 3
7. Music, dancing and gambling were 3) 2 and 3 4) 1,2 and 3
forbidden. Prostitution was strictly Answer: 4
prohibited. Explanation:
8. He abolished the practice of observing 1. Abu'l Rayan Alberuni was a philosopher
Navroz, the Persian New Year day. scientist, whose Kitab al Hind was the first
9. Jharokdia-i-Darslum or the practice of and most important discussion on Indian
addressing the public at the balcony of the sciences, religion and society by an
emperor's forts was banned. outsider.
10.He stopped the practice of weighing the 2. He was not just a historian.
emperor in silver and gold. 3. His knowledge and interest covered many
11.He dismissed royal astronomers and other areas such as astronomy, geography,
astrologers. logic, medicine, mathematics, philosophy,
12.Measures were adopted to control religion and theology.
fashions of the day. Strict orders were 4. He was originally from Khwarrazon and had
issued for women to observe purdah. Every served the last of the independent
Muslim was required to say prayers five Khwarazm Shah as a scholar and a diplomat
times a day and keep Ramadan fast. till Mahmud of Ghazni conquered this
kingdom.
a. After this, he was attached to Mahmud's
court and accompanied him to India during
17. “Fatwa-i-Alamgiri” written by Aurangazeb various raids.
deals with 5. Alberuni's Kitab al Hind or Tahqiq-i Hind is
1) Codification of unislamic Practices the survey of Indian life based on his study
2) Strategy on conquering Deccan states and observations in India between 1017
3) Rule book on succession and 1030.
4) Statecraft a. To get a proper grip of the situation, he
Answer: 4 learned Sanskrit so that he might go to the
sources of Hindu thought and religion.
Explanation:
b. He learnt Sanskrit to acquire first hand
 Fatwa-i-Alamgiri
information.

AMIGOS IAS 9
c. He read the religious texts and met the Shivamara I, numerous temples were
educated Indians. built. The Ganga general Chamundaraya
6. His approach was scientific and religious constructed the Colossal statue of
prejudices do not mar the quality of his Bahubali also known as
observations. Gommateshvara—the son of
a. He quoted from the Bhagavad Gita, Vishnu Rishabhanatha, the first Tirthankara at
Puran, Kapil's Sankhya and the work of Shravanabelagola.
Patanjali. 20. Consider the following regional states:
7. Alberuni's observation of Indian society are 1. Warangal 2. Jajnagar
related to: 3. Tirhut 4. Bengal
● Caste-ridden society Which sultan of Delhi annexed the above-
● Closed society mentioned regional states?
● Stagnant knowledge 1) Alauddin Khalji
● Social evils 2) Firuz Shah Tughluq
● Religious beliefs 3) Ghiyasuddin Tughlaq
● Scientific knowledge. 4) Muhammad Bin Tughluq
Hence, all the statements are correct. Answer: 3
19. The colossal Gomateshwara statue at Explanation:
Shravanabelagola was constructed with the Foreign Policy
patronage of  Ghiyasuddin pursued the policy of
1) The Hoysalas annexation. He annexed the territories of
2) The Western Gangas those rulers of the south which were
3) The Chalukyas of Badami tributary states under Alauddin.
4) The Rashtrakutas 1. Annexed Warangal:
Answer: 2 a. Prataprudra Deva, the ruler of Warangal
Explanation: had stopped the annual tribute.
Jainism b. Ghiyasuddin sent prince Jauna khan or
Ulugh Khan to annex the Kakatiya kingdom.
 During the early mediaeval period from
the 8th-12th Century, Jainism faded in the c. Prince Jauna besieged the fort of Warangal
northern parts of India but continued to in 1321 CE.
flourish in the south. In the south mainly d. Prataprudra Deva surrendered and
in the Karnataka region, the Digambara Telengana was annexed to Delhi Sultanate.
Jainism prospered and reached to its great e. The name of Warangal was changed to
heights. It received enormous patronage Sultanpur.
from three major Karnataka based 2. Annexed Jajnagar, Orissa:
dynasties of the early medieval period: a. Jauna Khan next attacked the kingdom of
1. The Gangas Jajnagar in Orissa.
2. The Rashtrakutas b. Its ruler Bhanudeva II, the ruler of Jajnagar
3. The Hoysalas had supported the ruler of Warangal
 Prominent Jain monks acted as advisors to against the sultan.
rulers and helped in their successions. The c. In 1324, Jauna Khan after plundering
rulers patronised the Jain community by Jajnagar annexed it to the Delhi Sultanate.
conferring land grants on them for 3. Annexed Parts of Bengal:
construction of Jain temples, pillars and a. In 1323-24 CE as a response to the request
monasteries. The Jains received the by nobles of Firuz Shah, the independent
highest patronage from the Western ruler of Lakhnauti, Ghiyasuddin marched to
Ganga rulers. It was under Ganga ruler Lakhnauti in Bengal and defeated its ruler.

AMIGOS IAS 10
b. East and south Bengal were annexed to the 3. The swadeshi movement failed to enlist
Sultanate. the cooperation of the Muslim masses of
4. Annexed Tirhut, Bihar: Bengal for various reasons.
a. On his way back from Bengal, Ghiyas-ud- a. The great majority of Muslims of Eastern
din defeated the Raja of Tirhut in north Bengal felt rightly or wrongly that they had
Bihar and annexed the region into the a greater prospect of material
sultanate. advancement in the newly created
21. Consider the following statements about province of East Bengal and Assam.
Swadeshi Movement: b. The boycott of British goods directly hit the
1. It had wider popular appeal in the History common Muslim consumers by raising the
of National movement prices of such essential commodities like
2. It had an economic character in Bengal only cloth, shoes and soap while there was no
Muslim trading class in Bengal which could
3. It enlisted the cooperation of the Muslim
take advantage of the swadeshi slogans.
Masses of Bengal
c. Some leaders of the swadeshi movement
Which of the statements given above are
like Bipin Chandra Pal, Aswini Kumar Datta,
correct ?
Surendranath Banerjea and Rabindranath
1) 2 only 2) 1 and 2
Tagore of course sought to win over the
3) 2 and 3 4) 1,2 and 3 Muslims to their side, but their success was
Answer: 1 limited.
Explanation: d. Ghaznavi, a Muslim zamindar and lawyer,
Character of Swadeshi Movement: took part in the Swadeshi Movement and
1. The Swadeshi Movement was more led the boycott of British leather goods.
spontaneous in character and had a wider e. Some common Muslims also took part in
popular appeal than any other political the Anti-Partition meetings.
agitation preceding it. Hence, statement 1 f. But very few of them did so with
is incorrect. conviction.
a. It was little dependent on the support and g. Not a single Muslim organisation officially
encouragement of the oficial leaders of the condemned the Partition of Bengal.
Congress. h. Unfortunately, there also occurred some
b. This was due to the fact that the discontent Hindu-Muslim clashes in East Bengal during
in Bengal was not confined to the educated the swadeshi agitation.
urban middle class but had penetrated, to i. Hence, statement 3 is incorrect.
some extent, into the rural society as well.
22. The Muslim Deputation to Lord Minto in 1906
2. The Anti-Partition agitation had an was led by
economic character in Bengal unlike the
1) Sayyid Ahmad Khan
revolutionary agitation in Maharashtra
2) Muhammad Ali Jinnah
which had a religious character.
3) Nawab Mohsin-ul-Mulk
a. Most of the leaders of the agitation in
Maharashtra were Chitpavan Brahmins. 4) Aga Khan
b. The programme of swadeshi and boycott Answer: 4
revealed the economic character of the Explanation:
Anti-Partition agitation. 1. Nawab Mohsin-ul-Mulk, who had
c. The boycott of British goods and the 'buy succeeded Sayyid Ahmad Khan as the
swadeshi' slogan undoubtedly gave a great leader of the Aligarh Movement decided to
stimulus to indigenous industries. send a Muslim deputation to the Viceroy
d. Hence, statement 2 is correct. for impressing upon him the necessity of
safeguarding the rights and interests of the
Muslim community.

AMIGOS IAS 11
2. The deputation was prompted by the realise that in politics they had no separate
decision of the British Government to interests as Hindus or Muslims."
introduce representative principles in the 7. The Lucknow kept the germs of
legislative councils of India. communalism in Indian Politics alive
3. This deputation which met the Viceroy in Hence, all the statements are correct.
Simla in October, 1906 was composed of 24. With respect to Mahatma Gandhi, Consider
35 persons, nobles, landowners, the following statements:
merchants and other loyal Muslim subjects 1. He was put on trial for four inflammatory
of the British Crown and it was led by Aga articles against the British in Young India
Khan, the Nawab of Dacca
2. There was no defence counsel to defend
23. Consider the following statements about him
Lucknow Pact:
Which of the statements given above is/are
1. Bal Gangadar Tilak played a very important correct ?
role in uniting the congress and the Muslim
1) 1 only 2) 2 only
League
3) Both 1 and 2 4) Neither 1 nor 2
2. It kept the germs of communalism in Indian
Answer: 3
Politics alive
Explanation:
Which of the statements given above is/are
correct ? 1. Trial of Mahatma Gandhi
1) 1 only 2) 2 only a. along with the publisher of Young India,
Shankarlal Banker
3) Both 1 and 2 4) Neither 1 nor 2
b. Put on Trial in 1922 for four inflammatory
Answer: 3
articles against the British in Young India
Explanation:
c. there was no defence counsel or legal
1. At the Lucknow Session of the Congress,
advice
the Congress and the Muslim League
d. sentenced to jail for six years (beginning
brushed aside their old differences and put
1922) but set free in February, 1924.
forward common demands before the
government. Hence, all the statements are correct.
2. The new political atmosphere created by 25. Who among the following described the
the war and the two Home Rule Leagues great revolt of 1857 as ‘the first struggle for
fostered this development to some extent. Indian Independence’ ?
3. The unity between the Congress and the 1) Jayaprakash Narayan
League was brought about by the signing of 2) Percival Spear
the Congress-League pact, popularly 3) William Dalrymple
known as the Lucknow Pact. 4) V D Savarkar
4. Tilak and Annie Besant played a very Answer: 4
important role in uniting the two. Explanation:
5. The two organisations passed the same  The Revolt of 1857, though a failure,
resolutions, put forward a joint scheme of continued to inspire the Indian
political reforms based on separate nationalists and revolutionaries of the
electorates and demanded self- later generation. Herein lies the
government. nationalist significance of the uprising.
6. Bipan Chandra has remarked, Savarkar drew inspiration from the Great
"Unfortunately, it was based on the notion Revolt when he described it as 'the first
of bringing together the educated Hindus struggle for Indian independence.'
and Muslims as separate entities; in other 26. Which of the following books were written by
words without secularisation of their B.R.Ambedkar ?
political outlook which would make them 1. Buddha and His Dhamma

AMIGOS IAS 12
2. The Evolution of Provincial Finance in Report the Government of India Act of
British India 1919 was passed.
3. Pakistan or Partition of India 1. By this legislation the Provincial Legislative
4. Thoughts on Linguistic States Councils were enlarged and the majority of
Select the correct answer using the codes their members were to be elected.
given below: 2. The provincial governments were given
1) 1 and 4 2) 2 and 3 more powers under the system of dyarchy.
3) 1,3 and 4 4) All of the above 3. Under dyarchy, the powers of provincial
Answer: 4 governments were to be divided into
'reserved' and 'transferred' subjects.
Explanation:
a. The reserved subjects which included
Some of his famous books include
essential areas of law enforcement such as
○ The Untouchable: Who are They and Why
justice, police and revenue, would be
They Have Become Untouchables
looked after by the Governor and two
○ Buddha and His Dhamma executive councillors appointed by the
○ ‘The Rise and Fall of Hindu Women‘ Crown.
○ ‘Emancipation of Untouchables’, b. In these subjects the people were not to
○ ‘The Evolution of Provincial Finance in have any say. But in matters relating to
British India’; 'transferred subjects' the people were to
○ ‘Pakistan or Partition of India‘, have a say.
○ Thoughts on Linguistic States’, etc. c. These included education, public works,
Hence, all the statements are correct. public health etc.
27. Consider the following statements about the 4. These Departments were under the
Government of India Act, 1919: ministers responsible to Provincial
1. The majority of members in provincial Legislative Councils.
legislative councils were to be elected. a. Since the members of these Councils would
2. It introduced the system of Dyarchy be elected by the Indian people, the people
3. Items like justice, police and revenue were had an indirect control on these
kept as reserved subjects Departments. While some of the spending
departments were transferred, the
4. Items like education, public works, public
Governor retained complete control over
health were kept as transferred subjects
the finances.
Which of the statements given above are
5. At the centre, the lower house or the
correct ?
Legislative Assembly, was to be manned by
1) 1,2 and 3 2) 2,3 and 4
41 nominated members out of the total
3) 1,3 and 4 4) All of the above number of 144 members.
Answer: 4 6. The Council of State, or the Upper house,
Explanation was to be manned by 26 nominated and 34
 The Government of India Act (1919) or the elected members.
Montagu-Chelmsford Report 7. The legislature could not exert any control
 In 1917 Edwin Montagu, the Secretary of over the Governor-General and his Council.
State, came to India and embarked on a 8. The Central Government had unrestricted
fact finding tour. He was accompanied by control over the provincial governments.
the Viceroy, Lord Chelmsford. The Hence, all the statements are correct.
following year, the two issued a report on 28. Consider the following statements about
Indian Constitutional Reforms. This is Rowlatt Act:
known as the Montagu-Chelmsford or
1. It was aimed at curtailing the civil liberties
Montford Report. On the basis of this
of the Indians

AMIGOS IAS 13
2. Jinnah and Madanmohan Malavya 29. Who was the Viceroy that held the Imperial
resigned from the legislative council in Durbar at Delhi in 1877 at a time when the
protest against it country was hit by a terrible famine ?
Which of the statements given above is/are 1) Lord Lytton 2) Lord Curzon
correct ? 3) Lord Harding II 4) Lord Ripon
1) 1 only 2) 2 only Answer: 1
3) Both 1 and 2 4) Neither 1 nor 2 Explanation:
Answer: 3  Regime of Lord Lytton (1876-1880)
Explanation:  The holding of the Imperial Durbar at
Rowlatt Act (1919) and the Rowlatt Delhi in 1877 at a time when the country
Satyagraha was hit by a terrible famine triggered off
1. Rowlatt Act (1919) was a repressive an adverse reaction among the Indians.
measure passed in March 1919. 30. Consider the following statements about The
2. This was the Rowlatt Act which was Ilbert Bill :
opposed by every single Indian member of 1. The legislation was passed during the
the Central Legislative Council. viceroyalty of Lord Ilbert.
3. The British Indian Government was 2. It enables the Indian district magistrates to
determined to arm itself with more try Europeans in criminal cases
coercive powers. 3. The Indian reaction to it was spearheaded
4. The Rowlatt Act of 1919 actually aimed at by Surendranath Banerjea
curtailing the civil liberties of the Indians in 4. The European reaction to it was
the name of crackdown on terrorist spearheaded by Branson.
violence. Which of the statements given above are
5. The Rowlatt Act was taken as a great blow correct ?
to the honour of the Indian nation. 1) 1,2 and 3 2) 2,3 and 4
6. The Act was particularly attacked because 3) 1,3 and 4 4) All of the above
of its timing as it came at the end of the
Answer: 2
war, when Indians were expecting
Explanation:
substantial constitutional concessions.
1. The Indian Association played a laudable
7. The Act sparked off a movement of protest.
role during the Ilbert Bill agitation.
8. Even moderates like Surendranath
2. In 1883, Lord Ripon sought to pass a
Banerjea recorded their reservations about
legislation to enable Indian district
the Act.
magistrates and Session Judges to try
9. Jinnah, Madanmohan Malavya and others
Europeans in criminal cases. Hence,
resigned from the legislative council in
statement 1 is incorrect. If you eliminate
protest against the Rowlatt Act.
this option you will arrive at the answer.
10.The Rowlatt Act was described by
3. Under the prevalent law even the Indian
contemporary media as "a gigantic
ICS officials were not empowered to try
blunder".
Europeans.
11.To launch a strong movement against the
4. It was an attempt to remove some
measure, a Satyagraha Sabha was formed
elements of racial discrimination.
under the leadership of Gandhi.
5. The Europeans in India banded together
12.The Satyagraha Sabha decided to stage civil
against the Bill, which came to be known
disobedience movement in India
after Ilbert, the Law Member of the Council
Hence, all the statements are correct. .

AMIGOS IAS 14
a. They were of opinion that even the most fractionated plateau and a flank zone all
highly educated among the Indians were along its length. The rift system at the
unfit to try an European crest is the zone of intense volcanic
6. Indian public opinion was quite formidable. activity. (Hence, the option 3 is the correct
They were annoyed at the racial bitterness option here)
displayed by the Europeans. They 32. Consider the following minerals
organised a counter campaign. i) Talc ii) Gypsum
7. The Indian reaction was spearheaded by iii) Calcite iv) Fluorite
Surendranath Banerjea and Lalmohan Arrange the minerals given above in
Ghose of the Indian Association. descending order of the degree of their
8. The European community, led by Mr. hardness?
Branson, a Barrister, even plotted to 1) i<ii<iii<iv 2) iv<iii<ii<i
kidnap Lord Ripon and deport him to 3) ii<iii<iv<I 4) ii<iii<i<iv
England.
Ans: 2
9. The Indian Association organised meetings
 The Hardness — relative resistance being
and demonstrations in support of the Bill.
scratched; ten minerals are selected to
10.Lord Ripon eventually had to surrender to measure the degree of hardness from 1-
European pressure and the jurisdiction of 10. They are: 1. talc; 2. gypsum; 3. calcite;
only the District Magistrates and the 4. fluorite; 5. apatite; 6. feldspar; 7.
Session Judges was acknowledged in quartz; 8. topaz; 9. corundum; 10.
principle. diamond. Compared to this, for example,
11.This was another humiliating defeat for the a fingernail is 2.5 and a glass or knife blade
Indians, who lost their struggle for equality. is 5.5. (Hence, the option 2 is the correct
12.The Ilbert Bill agitation however, was a option here)
significant chapter in the history of India's 33. Depending upon irregularities of the land
struggle for freedom. surface, the overland flow may concentrate
a. It taught the Indians the value of into narrow to wide paths, the Overland flow
combination and solidarity and made them causes which among the following erosion?
determined to oust their racist rulers from 1) Sheet erosion 2) Rill erosion
the Indian soil 3) Gully erosion 4) Flat erosion
Hence, statements 2,3 and 4 are correct. Ans: A
31. Consider the following description
 Overland flow causes sheet erosion.
This forms an interconnected chain of Depending upon irregularities of the land
mountain systems within the ocean. It is the surface, the overland flow may
longest mountain chain on the surface of the concentrate into narrow to wide paths.
earth though submerged under the oceanic (Hence, the option 1 is the correct option
waters. The description given above is here)
associated with which among the following?
 Because of the sheer friction of the
1) Exogenic landforms 2) Tillite landforms column of flowing water, minor or major
3) Mid-Oceanic Ridges 4) Abyssal plains quantities of materials from the surface of
Ans: 3 the land are removed in the direction of
 Mid-Oceanic Ridges: This forms an flow and gradually small and narrow rills
interconnected chain of mountain will form.
systems within the ocean. It is the longest 34. Match the following regarding the various
mountain chain on the surface of the forms of the erosional landforms
earth though submerged under the Name of the Formation
oceanic waters. It is characterised by a erosional landform
central rift system at the crest, a

AMIGOS IAS 15
i) Canyons Form in horizontally  The river finally discharges itself into the
bedded sedimentary Bay of Bengal near the Sagar Island.
rocks. 36. Consider the following statements
ii) Gorges Form in soft rocks. Statement (I):
iii) Plunge pools Deep holes at the EI-Nino is merely an extension of the cold
base of waterfalls. equatorial current that gets replaced
Which of the pairs given above is/are matched temporarily by a cold Peruvian current or
properly? Humbolt current.
1) Only I and ii 2) Only ii and iii Statement (II):
3) Only I and iii 4) All the above EI-Nino is a complex weather system that
Ans: 3 appears once every three to seven years,
bringing drought, floods and other weather
 The valleys start as small and narrow rills;
extremes to different parts of the world.
the rills will gradually develop into long
Select the answer using the codes given below
and wide gullies; the gullies will further
deepen, widen and lengthen to give rise to 1) Both Statement (I) and Statement (II) are
valleys. correct and Statement (II) is the proper
explanation of Statement (I).
 The canyons commonly form in horizontal
bedded sedimentary rocks and gorges 2) Both Statement (I) and Statement (II) are
form in hard rocks. (Hence, the pair I is correct and Statement (II) is not a proper
matched properly and the pair is not explanation of Statement (I).
matched properly) 3) Only Statement (I) is correct
 The large and deep holes at the base of 4) Only Statement (II) is correct
waterfalls are called plunge pools. (Hence, Ans: 4
the pair iii is matched properly)  The word EI-Nino means ‘Child Christ’
35. Consider the following tributaries because this current appears around
i) Ghaghara ii) Gandak Christmas in December. EI-Nino is a
iii) Kosi iv) Mahananda complex weather system that appears
once every three to seven years, bringing
The important left-bank tributaries of the
drought, floods and other weather
Ganga River are?
extremes to different parts of the world.
1) Only i and ii 2) Only i, ii, and iii (Hence, the statement (II) is correct here)
3) Only ii and iii 4) All the above
 The system involves oceanic and
Ans: 4 atmospheric phenomena with the
 The Ganga is the most important river of appearance of warm currents off the coast
India both from the point of view of its of Peru in the Eastern Pacific and affects
basin and cultural significance. It rises in weather in many places including India.
the Gangotri glacier near Gaumukh (3,900  EI-Nino is merely an extension of the
m) in the Uttarkashi district of warm equatorial current which gets
Uttarakhand. Here, it is known as the replaced temporarily by a cold Peruvian
Bhagirathi. It cuts through the Central and current or Humbolt current. This current
the Lesser Himalayas in narrow gorges. increases the temperature of water on the
 The Son is its major right bank tributary. Peruvian coast by 10°C. Hence, the
The important left bank tributaries are the statement (I) is incorrect here)
Ramganga, the Gomati, the Ghaghara, the 37. The Blossom Shower is a type of shower that
Gandak, the Kosi and the Mahananda. causes coffee flowers to bloom in nearby
(Hence, the option 4 is the correct option areas, these showers are common in which
here) among the following states?

AMIGOS IAS 16
1) West Bengal 2) Tamil Nadu Which of the districts in Andhra Pradesh have
3) Kerala 4) Karnataka the boarder with the neighbouring state of
Ans: 3 Telangana?
 The Blossom Shower makes coffee flowers 1) Only i and ii 2) Only i, ii, and iii
blossom in Kerala and nearby areas. 3) Only i, ii, and iv 4) All the above
 In the heart of the ITCZ in the northwest, Ans: 2
the dry and hot winds known as ‘Loo’,  Seven districts in Andhra Pradesh have a
blow in the afternoon, and very often, border with the neighbouring state of
they continue to well into midnight. Dust Telangana, they are:
storms in the evening are very common  Alluri Seetharama Raju, Eluru, NTR,
during May in Punjab, Haryana, Eastern Palnadu, Prakasam, Nandyala, and
Rajasthan and Uttar Pradesh. Kurnool. (Hence, option 2 is the correct
38. Consider the following statements regarding option here)
the Coriolis force 40. Match the following regarding the state
i) It is the force that the rotation of the earth symbols of Andhra Pradesh
about its axis affects the direction of the Name State symbol
wind. i) State animal Blackbuck
ii) It deflects the wind to the left direction in ii) State bird Palapitta
the northern hemisphere and the right in iii) State tree Vepa Chettu
the southern hemisphere.
iv) State flower Tamara puvvu
iii) It is maximum at the equator and is absent
Which of the pairs given above is/are not
at the poles.
matched properly?
Which of the statements given above is/are
1) Only i and ii 2) Only i, ii, and iii
correct?
3) Only ii and iv 4) None of the above
1) Only i 2) Only ii and iii
Ans: 3
3) Only i and iii 4) All the above
STATE ANIMAL: BLACK BUCK (Antilope
Ans: 1
cervicapra)
 The rotation of the earth about its axis
 The blackbuck (Krishna Jinka) is a diurnal
affects the direction of the wind. This
antelope and inhabits grassy plains and
force is called the Coriolis force after the
slightly forested areas. Due to their
French physicist who described it in 1844.
regular need for water, they prefer areas
(Hence, the statement 1 is correct here)
where water is perennially available.
 It deflects the wind to the right direction STATE BIRD ROSE RINGED PARAKEET
in the northern hemisphere and the left in (Psittacula krameri)
the southern hemisphere. The deflection
 The rose-ringed parakeet (Rama Chiluka)
is more when the wind velocity is high.
also known as the ring-necked parakeet, is
(Hence, the statement ii is incorrect here)
a medium-sized parrot and it is sexually
 The Coriolis force is directly proportional dimorphic. The adult male sports a red
to the angle of latitude. It is maximum at and black neck ring, and the hen and
the poles and is absent at the equator. immature birds of both sexes either show
(Hence, the statement iii is incorrect here) no neck rings or display shadow-like pale
39. Consider the following districts of Andhra to dark grey neck rings.
Pradesh
STATE TREE NEEM (Azadirachta indica)
i) Eluru
 Neem (Vepa Chettu) is native to the Indian
ii) Alluri Seetharama Raju subcontinent. It is typically grown in
iii) Prakasam tropical and subtropical regions.
iv) West Godavari STATE FLOWER JASMINE (Jasminum officinale)

AMIGOS IAS 17
 Jasmine (Malle Puvvu) is a genus of shrubs areas during winters is known as
and vines in the olive family (Oleaceae). transhumance.
Jasmines are widely cultivated for the  In mountain regions, such as the
characteristic fragrance of their flowers. Himalayas, Gujjars, Bakarwals, Gaddis and
Jasmine can be either deciduous or Bhotiyas migrate from plains to the
evergreen and can be erect, spreading, or mountains in summers and to the plains
climbing shrubs and vines. from the high-altitude pastures in winters.
41. Consider the following minerals Similarly, in the tundra regions, the
i) platinum ii) zinc nomadic herders move from south to
iii) tin iv) aluminium north in summer and from north to south
Which of the above comes under the non- in winter. (Hence, the option 4 is the
ferrous metals group? correct option here)
1) Only i, ii, and iii 2) Only ii, iii, and iv 43. Consider the following description
3) Only iii and iv 4) All the above This type of agriculture is characterised by the
dominance of the rice crop and the Land
Ans: 2
holdings are very small due to the high density
 The basic source of all minerals is the hot of population, this description is associated
magma in the earth's interior. When with which of the following farming?
magma cools, crystals of minerals appear
1) Primitive subsistence agriculture
and a systematic series of minerals are
formed in sequence to solidify to form 2) Intensive Subsistence Agriculture
rocks. Minerals such as coal, petroleum 3) Commercial horticulture Agriculture
and natural gas are organic substances in 4) None of the above
solid, liquid and gaseous forms Ans: 2
respectively.  Intensive subsistence agriculture
 Non-ferrous metals: include metals like dominated by wet paddy cultivation: This
copper, lead, zinc, tin, aluminium etc. type of agriculture is characterised by the
Here platinum comes under the precious dominance of the rice crop. Land holdings
metals category. (Hence, the option 2 is are very small due to the high density of
the correct option here) population. Farmers work with the help of
42. Consider the following tribes family labour leading to intensive use of
i) Bakarwals land. Use of machinery is limited and most
ii) Gaddis of the agricultural operations are done by
manual labour. Farmyard manure is used
iii) Bhotiyas
to maintain the fertility of the soil. In this
Which of the pastoral tribal groups of the type of agriculture, the yield per unit area
Himalayas migrate from the plains to the is high but per labour productivity is low.
mountains in summer and to the plains from (Hence, the option 2 is the correct option
the high-altitude pastures in winter? here)
1) Only i 2) Only ii and iii 44. In this method, vertical deep bores have to be
3) Only i and ii 4) All the above sunk, from where underground galleries
Ans: 4 radiate to reach the minerals, this type of
 The Movement in search of pastures is mining is known as?
undertaken either over vast horizontal 1) Opencast mining 2) Quarrying
distances or vertically from one elevation 3) Shaft mining 4) Drilling
to another in the mountainous regions. Ans: 3
The process of migration from plain areas
 Depending on the mode of occurrence
to pastures on mountains during summers
and the nature of the ore, mining is of two
and again from mountain pastures to plain
types: surface and underground mining.

AMIGOS IAS 18
Surface mining also known as open-cast production and efficiency. Formulate a
mining is the easiest and the cheapest way coking coal strategy to enhance coking
of mining minerals that occur close to the coal availability in the country. Improve
surface. Overhead costs such as safety the quality of coal
precautions and equipment are relatively  Achieve CAPEX target of Rs 21030 Crores
low in this method. for FY 2023-24. Monetize assets. Allocate
 The output is both large and rapid. When 25 new coal mines for commercial mining
the ore lies deep below the surface, the in FY 2023-24.
underground mining method (shaft 46. The oil extracted from the wells is crude oil
method) has to be used. In this method, and contains many impurities, it cannot be
vertical shafts have to be sunk, from used directly. It needs to be refined, which
where underground galleries radiate to among the following is an example of a
reach the minerals. (Hence, the option 3 is market-based refinery in India?
the correct option here) 1) Barauni 2) Digboi
 Minerals are extracted and transported to 3) Ankaleshwar 4) None of the above
the surface through these passages. It Ans: 1
requires specially designed lifts, drills,
 Crude petroleum consists of
haulage vehicles, and ventilation systems
hydrocarbons of liquid and gaseous states
for the safety and efficient movement of
varying in chemical composition, colour
people and materials. This method is risky.
and specific gravity. It is an essential
Poisonous gases, fires, floods and caving
source of energy for all internal
in lead to fatal accidents.
combustion engines in automobiles,
45. Assertion (A): railways and aircraft. Its numerous by-
India ranked first in coal production. products are processed in petrochemical
Reason (R): industries, such as fertiliser, synthetic
Coal accounts for almost 50 percent share in rubber, synthetic fibre, medicines,
the total installed electricity generation Vaseline, lubricants, wax, soap and
capacity in India. cosmetics.
Select the answer using the codes given below  Oil extracted from the wells is crude oil
1) Both A and R are true, and R is the proper and contains many impurities. It cannot be
explanation of A used directly. It needs to be refined. There
2) Both A and R are true, and R is not a proper are two types of refineries in India: (a)
explanation of A field-based and (b) market-based. Digboi
3) Only A is true is an example of field-based and Barauni is
4) Only R is true an example of a market-based refinery.
(hence, option 1 is the correct option
Ans: 4
here)
 India has the fifth largest coal reserves, is
47. Consider the following elements
second in coal production (after China)
i) Uranium
and is among the largest importers of coal
(mainly from Indonesia, South Africa and ii) thorium
Australia). (Hence, the Assertion (A) is iii) potassium
incorrect here) Which of the elements given above
 Coal accounts for almost 50% share in the contributes to natural terrestrial
total installed electricity generation radioactivity?
capacity in India. (Hence, the Reason is 1) Only i 2) Only i and ii
correct here) 3) Only ii 4) All the above
 Achieve the coal production target of Ans: 4
1012 MT for FY 2023-24. Increase coal

AMIGOS IAS 19
 Uranium is a silvery-grey metallic iii) The world’s largest Integrated Renewable
radioactive chemical element. It is only Energy Storage Project is being developed
naturally formed in supernova explosions. in the state and is scheduled to be
Uranium, thorium, and potassium are the commissioned by the end of 2025.
main elements contributing to natural Which of the statements given above is/are
terrestrial radioactivity. (Hence, the correct?
option 4 is the correct option here) 1) Only i and ii 2) Only ii
 Uranium has the chemical symbol U and 3) Only ii and iii 4) All the above
atomic number 92. Ans: B
 Uranium isotopes in natural uranium are  The state government has come up with
238U (99.27%) and 235U (0.72%). the ‘Andhra Pradesh Renewable Export
 All uranium isotopes are radioactive and Policy -2020’ to establish 120 GW capacity
fissionable. But only 235U is fissile (which renewable power projects by 2025. The
will support a neutron-mediated chain state is home to 4 solar parks, which can
reaction). Traces of Uranium are found accommodate 4 GW of capacity. The
everywhere. Commercial extraction is World’s largest Integrated Renewable
possible only in locations where the Energy Storage Project (IRESP), with a
proportion of Uranium is adequate. There capacity of 5,230 MW is being developed
are very few such locations. in the state and is scheduled to be
48. Consider the following crops commissioned by the end of 2023. The
i) rice state has the potential to become Green
ii) sugarcane Hydrogen for the global market. IH2A
iii) jute (Indian Hydrogen Alliance), has proposed
five green hydrogen hubs in India to be
iv) soya
developed by 2025, with capacity of 150
Which of the above are water-intensive crops?
GW of electrolyser capacity. Out of these
1) Only I and ii five hubs two are proposed in Andhra
2) Only I, ii, and iii Pradesh at Visakhapatnam (30 MW) and
4) Only I, ii, and iv Krishnapatnam (30 MW).
4) All the above 50. According to the AP Socio-Economic Survey
Ans: 4 2022-23, the total number of inter-state
 The Water need of certain crops also rivers in Andhra Pradesh is?
makes irrigation necessary. For instance, 1) 25 2) 11
the water requirement of rice, sugarcane, 3) 15 4) 21
jute, soya etc. is very high which can be Ans: 3
met only through irrigation. These crops  Besides prominent rivers like Godavari,
are also known as water-intensive crops. Krishna, Pennar, Vamsadhara and
(hence, option 4 is the correct option Nagavali, there are 40 major, medium and
here) minor rivers in the state of Andhra
49. Consider the following statements regarding Pradesh. Of the 40 rivers, 15 are inter-
the state rivers which are: I.Bahuda River (AP
i) The aim of Andhra Pradesh Renewable & Orissa), 2.Mahendra Tanaya River (AP &
Export Policy -2020 is to establish 120 GW Orissa), 3. Poondi Minor Drainages (AP &
capacity renewable power projects by Orissa), 4. Vamsadhara River (AP &
2030. Orissa), 5.Nagavali River (AP & Orissa),
ii) The State of Andhra Pradesh is home to 4 6.Godavari (AP, Maharashtra, Orissa,
solar parks which can accommodate 4 GW Karnataka, Chattisgarh & Madhya
of capacity. Pradesh), 7.Krishna (AP, Maharashtra&

AMIGOS IAS 20
Karnataka), 8. Pennar River (AP & handicraft products, and to provide space
Karnataka), 9. Araniar River (AP & Tamil for such products from all other States.
Nadu), 10.Kortaliar River (AP & Tamil Which of the statements given above is/are
Nadu), 11. Palar River (AP, Karnataka correct?
&Tamil Nadu), 12.Ponniar River (AP, 1) Only i 2) Only ii
Karnataka &Tamil Nadu), 13.Budameru 3) Both 4) None
(AP, Telangana), 14.Errakalava (AP,
Ans: 3
Telangana), 15.Tammileru (AP
,Telangana). (Hence, option 3 is the  The setting up of Ekta/Unity Mall in the
correct option here) States was announced in the Union
Budget 2023-24 for the promotion and
51. Consider the following physical factors
sale of their ODOPs, GI products, and
i) climate other handicraft products, and for
ii) terrain providing space for such products in all
iii) availability of water other States. The Union Budget also
Which of the above physical factors provided for an outlay of Rs. 5,000 crores
determines the pattern of the population of fifty-year interest-free loans to States
distribution? under the ‘Scheme for Special Assistance
1) Only I and ii 2) Only ii and iii to States for Capital Investment 2023-24’,
3) Only I and iii 4) All the above which will be linked to or allocated for
Ans: 4 certain purposes which, inter-alia,
includes constructing the Unity Malls. At
 As far as the physical factors are
present, 27 States have submitted their
concerned, it is clear that climate along
Detailed Project Reports, out of these, 17
with terrain and availability of water
have been approved by the Department of
largely determines the pattern of the
Expenditure.
population distribution. (Hence, the
option 4 is the correct option here) 53. Consider the following factors
 Consequently, we observe that the North i) Death ii) Birth
Indian Plains, deltas and Coastal Plains iii) Fertility iv) Migration
have a higher proportion of the v) Mortality
population than the interior districts of Which of the factors is not a component of
southern and central Indian States, the population change?
Himalayas, and some of the northeastern 1) Only I, ii, and iii 2) Only iii and v
and the western states. However, the 3) Only iii and iv 4) All the above
development of irrigation (Rajasthan), Ans: 2
availability of mineral and energy
 There are three components of
resources (Jharkhand) and development
population change – births, deaths and
of transport network (Peninsular States)
migration.
have resulted in moderate to high
concentration of population in areas that  Fertility and mortality are not components
were previously very thinly populated. of population change. (Hence, option 2 is
the correct option here)
52. Consider the following statements regarding
the Unity Mall 54. What is the rank of India in the World
Happiness Report 2023?
i) The setting up of Ekta/Unity Mall in the
States was announced in the Union Budget 1) 126 2) 134
2023-24. 3) 112 4) 96
ii) the main objective of the Unity Mall is to Ans: 1
promote and sale of their own One District  Recently, the UN Sustainable
One Product, GI products, and other Development Solutions Network released

AMIGOS IAS 21
the World Happiness Report 2023 which iii) Sometimes there is a negative population
ranks countries on happiness. Since 2012, growth rate
the World Happiness Report has been Which of the above are the main
released annually around March 20th as characteristics of the 4th stage of the
part of the International Day of Happiness demographic transition?
celebration. The report ranks global 1) Only i 2) Only i and iii
happiness based on survey data from 3) Only i and ii 4) All the above
people in over 150 countries.
Ans: 4
 The ranking uses six key factors to
 In 4th stage of demographic transition
measure happiness — social support,
both fertility and mortality decline
income, health, freedom, generosity, and
considerably. the population is either
absence of corruption. Besides ranking
stable or grows slowly. Sometimes there is
the countries, the report also looks at the
a negative population growth rate causes
state of the world in 2023.
the ageing of the population. The
 India ranks at 126th position out of 136 population becomes urbanised, literate
countries, making it one of the least happy has high technical know-how and
countries in the world. (Hence, the option deliberately controls the family size. This
1 is the correct option here) shows that human beings are extremely
55. Consider the following statements regarding flexible and can adjust their fertility.
the development (Hence, the option 4 is the correct option
i) Development means a qualitative change here)
which is always value positive. 57. Consider the following sectors
ii) Positive growth always leads to i) Specialists
development. ii) Research and development
Which of the statements given above is/are iii) Consultants
incorrect? iv) Policy formulations
1) Only i 2) Only ii Which of the sectors come under the quinary
3) Both 4) None sector?
Ans: 2 1) Only I and iii
 Development means a qualitative change 2) Only I, iii, and iv
that is always value positive. This means 3) Only I and iv
that development cannot take place
4) All the above
unless there is an increment or addition to
Ans: 2
the existing conditions. (The statement 1
is correct here)  The quinary sector includes specialists,
decision makers, policy formulations and
 Development occurs when positive
consultants included under quinary
growth takes place. Yet, positive growth
activity. (Hence, option 2 is the correct
does not always lead to development.
Development occurs when there is a option here)
positive change in quality. The statement 58. How many States have notified their
2 is correct here. respective State Logistics policies as of
December 2023?
56. Consider the following statements regarding
the demographic transition 1) 21 2) 26
i) both fertility and mortality decline 3) 23 4) 18
considerably. Ans: 3
ii) The population is either stable or grows  National Logistics Policy, 2022:
slowly  On completion of one year of the launch
of the National Logistics Policy, significant

AMIGOS IAS 22
progress has been made to achieve NLP 1) Gadaba 2) Porja
targets, viz. reduction in logistics cost, 3) Savara 4) Chenchu
improvement in India’s ranking in the Ans: 4
Logistics Performance Index (LPI), and  Chenchu tribe is the most vulnerable tribal
create data-driven decision support group in the State. They are still
mechanism for an efficient logistics dependent on hunting and food
ecosystem. gathering. They have not attained
 Progress made in the implementation of significant level of educational, economic
the Comprehensive Logistics Action Plan progress and are having low health
(CLAP) is as below: indices. (Hence, option 4 is the correct
 Infrastructure gaps are being addressed option here)
and digital initiatives undertaken (under  Recommendations:
the National Committee on Trade  Konda Reddy and Porja communities have
Facilitation). traditional skills in bamboo basketry and
 To bring a holistic focus on ‘logistics’ in jewellery making. Whereas Savara and
public policy at the State level, States/UTs Gadaba tribe mostly depend on collection
are developing State Logistics Plans (SLPs) of forest produce and agricultural
aligned with NLP. So far, 23 States have activities. Thus, special training could be
notified their respective State Logistics imparted to these tribal groups to
policies. (Hence, option 3 is the correct improve their traditional skills in
option here) agriculture, horticulture and micro
 Draft Sector Specific Plans developed by entrepreneurial activities.
M/o Coal and discussed in 6th EGoS. 61. In which of the following type of family, a
59. Which among the following districts in couple after marriage resides with the
Andhra Pradesh has the highest women groom’s maternal uncle’s house?
population? 1) Patrilocal family 2) Matrilocal family
1) Vishakhapatnam 2) Krishna 3) Bi-local family 4) Avunculocal family
3) East Godavari 4) Vijayanagaram Answer: 4
Ans: 3 Explanation:
 Andhra Pradesh is the tenth-largest state Based on residence, the family is classified as:
in the Country, in terms of population. As • Patrilocal/Virilocal family: It is the family
per the 2011 Census, the State accounts when the newly married wife leaves her
for 4.09 per cent of the total population of parents' house and goes to reside with her
the country. husbands' parents' house.
 The districts which have the highest • Matrilocal/Uxorilocal family: It is the
women population are family, when the newly married couple
 East Godavari (Hence, the option 3 is the resides in the wife's parents’ residence. Ex:
correct option here) Tribes of north-east such as Garo and
 Guntur Khasa.
 The least women population districts are • Bi-local/Ambilocal family: Follows both the
 Vijayanagaram and Srikakulam patrilocal and matrilocal. It is also known as
60. According to the tribal welfare department family of changing.
government of Andhra Pradesh, which • Avunculocal family: In this type a couple
among the following tribe is most vulnerable after marriage resides with the groom's
tribal group in the state of Andhra Pradesh mother's brother's family or maternal
and they still dependent on hunting and food uncles' house. Ex: Ashanti Ghana
fathering for their survival? 62. Levirate marriage is practiced among which
of the following tribe?

AMIGOS IAS 23
1) Toda 2) Gond • The 4 core principles of the Convention on
3) Chenchu 4) Erukali the Rights of the Child are:
Answer: 1 1. Non-discrimination
Explanation: 2. Right to life, survival and development
• Cross cousin marriage: It is the marriage of 3. Best interests of the child
a man with his maternal uncle's daughter 4. Respect for the child's views
of paternal aunt's daughter. It means 64. Consider the following statements
marriage of a girl with her mother's I. Sanskritization is a process of social
brother’s son or father's sister son. mobility.
• Levirate: It is the custom in which a widow II. This concept was developed by M N
marries her husbands' brother. Ex: Toda of Srinivas.
Nilgiris hills. Which of the above given statements is/are
• Sororate: It is the custom in which a correct?
widower marries the younger sister of his 1) I only 2) II only
deceased wife. Ex: Tribes of Gond or Baiga.
3) Both I and II 4) Neither I nor II
63. Which of the following is not a major
Answer: 3
category of right under the United Nations
Explanation:
Convention on the Rights of the Child?
• By definition, caste system is viewed as a
1) Right to Survival
rigid and closed system of stratification.
2) Right to Development
That is there is no movement or mobility
3) Right to Participation from one caste status to the other.
4) Right to Freedom • Social mobility is the process by which
Answer: 4 individuals or groups move, either upwards
Explanation: or downwards, from one social status to
Convention on the Rights of the Child (CRC) another in the social hierarchy.
• On November 20, 1989, the United Nations • In reality, social mobility has been an
General Assembly adopted the Convention important feature of caste system. This
on the Rights of the Child. It is a clearly means that caste system is a
comprehensive child rights convention. dynamic reality with flexibility in terms of
• The Convention on the Rights of the Child its structure and function.
was drafted in 1989 and came into force on • The concept of Sanskritization which is
September 02, 1990, while India signed it basically a process of social mobility was
on December 11, 1992. developed by M N Srinivas to describe the
Highlights of CRC: dynamic nature of the caste system.
• According to this all persons less than 18 Hence, statements I and II are correct.
years of age are children 65. Which Act treated tribes as “Backward
• The Convention provides for the rights of Tribes”?
the child through 52 articles under 1) Government of India Act, 1935
following four major categories: 2) Government of India Act, 1947
• Right to Survival - to life, health, nutrition, 3) Government of India Act, 1919
name, nationality 4) Councils Act, 1892
• Right to Development - to education, care, Answer: 1
leisure, recreation, cultural activities Explanation:
• Right to Protection - from exploitation, • The specification of tribes and tribal
abuse, neglect communities as Scheduled Tribes present
• Right to Participation - to expression, some problems. Even the social scientists
information, thought, religion have found it difficult to evolve a

AMIGOS IAS 24
universally acceptable definition for a • Since 1972, SEWA membership has grown
tribe. at a steady pace, drawing self-employed
• The difficulty in setting out formal criteria women from many different trades into
for' defining a tribe arises from the fact that the union: from vegetable vendors to
the tribes in India are, and have been for incense-stick rollers, from junksmiths to
some decades, tribes in transition. waste-recyclers. The Women’s Decade,
• The first serious attempt to list "primitive beginning in 1975 also gave a boost to the
tribes" was, as in the case r of depressed growth of SEWA, placing it firmly within the
castes, made at census of 1931: women’s movement. In 1977, SEWA’s
• Subsequently, under the Government of General Secretary, Ela Bhatt, was awarded
India Act, 1935, a list of "backward tribes" the prestigious Ramon Magsaysay Award
was specified for the Provinces of India. and this brought international recognition
to SEWA. Hence, statement II is correct.
• The list of Scheduled Tribes was prepared
in 1950 by making additions to the list of • By 1981, relations between SEWA and TLA
backward tribes under thc Government of had become strained. The interests of TLA,
India Act, 1935; in considering fresh representing workers in the organized
proposals for inclusion in the list, it was sector did not align easily with the interests
noted in drawing up the schedule of SEWA, representing unorganized
population were not at this stage women workers.
distinctiveness as tribes; and that 67. If two persons are descended from a common
communities which might be regarded as father but by different mothers, they are
tribes' organisation and general way of life called
but which were really not primitive should 1) Full blood relatives
not now newly be treated as primitive". 2) Half blood relatives
66. Consider the following statements regarding 3) Uterine blood relatives
the Self-Employed Women Association 4) Parallel cousins
I. It was founded by Ela Bhatt in 1972. Answer: 2
II. Ela Bhatt was awarded the prestigious Explanation:
Ramon Magsaysay Award. • Uterine: When descent is traced through
Which of the above given statements is/are female line.
correct? • Siblings: Children of the same parents
1) I only 2) II only • Full blood: Two persons are said to be
3) Both I and II 4) Neither I nor II related to each other by full blood when
Answer: 3 they are descended from a common
Explanation: ancestor by the same wife. (Kids with same
• SEWA was born as a trade union of poor parents)
self-employed women in 1972, in the city • Half-blood: Two persons are said to be
of Ahmedabad, Gujarat. It grew out of the related to each other by half-biood when
Women’s Wing of the Textile Labour they are descended from a common
association, TLA, India’s oldest and largest ancestor but by different wives. (Kids with
union of textile workers founded in 1920 same father but different mothers)
by Anasuya Sarabhai and Mahatma • Uterine blood: Two persons are said to be
Gandhi. The original purpose of the related to each other by uterine blood
Women’s Wing was to provide training in when they are descended from a common
sewing, spinning, knitting, embroidery, and ancestress but by different husbands. (Kids
other welfare activities to the wives and with same mother but with different
daughters of mill workers. Hence, father).
statement I is correct.

AMIGOS IAS 25
• Parallel Cousins/Kins: Children of siblings of • The Sarada Commission was set up and on
the same sex as children of brothers or the recommendations of this commission
children of sisters. Parallel cousins can be the Sarada Act of 1929 was enacted which
Matrilineal i.e. children of Mother's sister changed the age of marriage for girls to 14
(Chinnama) or Patrilineal i.e. children of years and the age of consent to 15 years
father's brother (Babai). and later on (in 1978) the age was raised to
• Cross Cousins/Kins: Children of siblings of 21 for boy and 18 for girl
opposite sex as children of brother(s) and • B M Malabari worked hard against child
sister(s). Cross cousins can be Matrilineal marriages among Parsis.
i.e. children of Mother's brother (Mama) or • The British banned polygamy in 1872 due
Patrilineal i.e. children of father's sister to the efforts of Keshab Chandra Sen.
(Athama). Women Remarriage Movements
• Lineal kinship or the direct line of • As a result of Child Marriages and
consanguinity is the relationship between Kanyashulkam, many become widows at
persons, one of whom is a descendant of an early age. They had no social and
the other. Examples are like from father to cultural rights. They were locked in a dark
son, grandfather to grandson etc room a forbidden to see light.
68. Consider the following pairs • Ishwar Chandra Vidyasagar is considered
Women issues Associated person as the Champion of Widow Remarriage.
I. Sati – Raja Ram Mohan Roy Due to his efforts, the Governor General
II. Child Marriage – Ishwar Chandra Vidya Dalhousie enacted the Widow Remarriage
Sagar Act in 1856 and legalized widow
III. Widow Remarriage – Keshab Chandra Sen remarriage. He conducted the First widow
marriage in India on December 27th, 1856
Which of the above given pairs is/are correctly
in Calcutta and his son was also married to
matched?
widow.
1) I only 2) I and II only
• Brahmo Samaj, Arya Samaj and Pradhana
3) II and III only 4) All the above
Samaj participated in this movement.
Answer: 1
• In Maharashtra this movement was led by
Explanation: Pandit Vishnu Shastri, Mahadev Govinda
• Raja Ram Mohan Roy, the founder of Ranade, RG Bhandarkar and DK Karve.
Brahmo Samaj, raised awareness among • D. K. Karve himself married a widow and
the people and convinced the then set up a Widows' Sadan in Poona. Later,
Governor General, William Bentick, then Jyotiba Phule and Bhagya Reddy Varma
British Government in 1829 brought 17th also fought for the rights of widow’s part of
Regulation banning Sati and thought the the backward class movements and built
practice was almost disappeared. houses for them to get education
• But in 1986 one incident of Sati employment and to provide shelter.
sahagamana happened in the state of 69. Consider the following statements
Rajasthan, then in 1987 the Indian
I. Negritos were the earliest people to have
Government enacted The Commission of
come to India.
Sati (Prevention) Act, 1987.
II. Austrics laid the foundation of Indian
Movements against Child Marriage
civilization.
• Raja Ram Mohan Roy, Kesava Chandrasen,
III. Mongoloids are found in the North-Eastern
Malabari, Harbilas Sharada etc.
part of India.
participated in the protests against child
Which of the above given statements is/are
marriage.
correct?
1) I only 2) I and II only

AMIGOS IAS 26
3) II and III only 4) All the above 4) Discontent Stage – Initiation Stage – Public
Answer: 4 Support Stage – Formalisation Stage
Explanation: Answer: 1
Ethnic Composition Explanation:
• India is a country with huge racial and Process of Growth of Agitation:
ethnic diversity. Indian people can be • There are some stages of youth unrest
divided into six main ethnic groups. These which finally turn into youth agitations.
include: These stages are:
• Negritos They are originally from Africa. • The Discontent Stage: The discontent stage
They were the earliest people to have is the stage of dissatisfaction and growing
come to India. They have survived in their confusion with the existing conditions.
original habitat in Andaman and Nicobar • The Initiation Stage: In the initiation stage
Islands. Hence, statement I is correct. a leader emerges who makes others
• Austrics They laid the foundation of Indian conscious of the causes of discontent. In
civilisation. They cultivated rice and this stage excitement increases and
vegetables and made sugar from proposals for actions are debated.
sugarcane. Now these people are found in • The Formalisation Stage: In this stage the
some parts of India, Myanmar and the programmes are developed, alliances are
islands of South-East Asia. Their languages forged, and support is also sought from the
have survived in the central and Eastern people concerned.
India. Hence, statement II is correct. • The Public Support Stage: In this stage the
• Mongoloids They are found in the North- problems of the youth are recognised as
Eastern part of India. Hence, statement III public problem. This not only creates
is correct. awareness among the public but also seeks
• Mediterranean or Dravidians They are the public support on the concerned issues.
people of South India. They have been 71. Which of the following pair is incorrect?
believed to come before the Aryans. They 1) Collateral joint family - Two or more
are reputed to have built up the city married couples between whom there is a
civilisation of the Indus valley. sibling bond live together
• Western Brachycephalics They include the 2) Supplemented collateral joint family -
broad headed people living mainly on the Collateral Joint Family + unmarried,
Western side of the country such as the divorced or widowed relatives
Ganga valley. 3) Lineal joint family - Two couples in which
• Nordics These were the last one to there is lineal link (parent & married
immigrate to India. They came to India son/married daughter) live together
somewhere between 2000 and 1500 BC. 4) Supplemented Lineal Joint Family - Three
They are now mainly found in the Northern or more couples are linked lineally and
and Central part of India. collaterally
70. Which of the following is the correct order of Answer: 4
stages in the process of growth of Youth
Explanation:
Agitation?
Types of Joint Family: -
1) Discontent Stage – Initiation Stage –
• According to Pauline Colenda, the
Formalisation Stage – Public Support Stage
following are the various types of Joint
2) Initiation Stage - Discontent Stage –
Families.
Formalisation Stage – Public Support Stage
• Collateral Joint Family: - Two or more
3) Initiation Stage – Discontent Stage – Public
married couples between whom there is a
Support Stage – Formalisation Stage
sibling bond live together. Ex: Two brothers

AMIGOS IAS 27
& their wives live together with their • The arrangement of marriage is largely the
unmarried children. responsibility of the parents, particularly
• Supplemented Collateral Joint Family: - that of the father.
Collateral Joint Family + unmarried, 73. Which of the pair is incorrect regarding the
divorced or widowed relatives. women legislations in India?
• Lineal Joint Family: - Two couples in which 1) Abolition of practicing Sathi – 1829
there is lineal link (parent & married 2) The Child Marriage Restraint Act – 1929
son/married daughter) live together. 3) The Dowry prohibition Act – 1961
• Supplemented Lineal Joint Family: - Lineal 4) The Protection of Women from Domestic
Joint Family + unmarried, divorced or Violence Act – 2006
widowed relatives who don't belong to Answer: 4
either of lineally linked families. Ex:
Explanation:
Father's widower brother
• Abolition of Practicing Sathi -1829
• Lineal Collateral Joint Family: Three or
• The Child Marriage Restraint Act (Sharada
more couples are linked lineally and
Act) - 1929
collaterally.
• Hindu Widow Remarriage Act - 1856
• Supplemented Lineal-Collateral Joint
Family: Lineal Collateral Joint Family + • The Maternity Benefit Act (Amended in
unmarried, widowed or separated relatives 2017) - 1961
who belong to either one of the lineal or • The Equal Remuneration Act - 1976
collateral nuclear families. Ex: Unmarried • The Immoral Traffic (Prevention) Act –
nephew of father. 1956
72. What are the essential elements of Muslim • The Dowry Prohibition Act - 1961
marriage? • Medical Termination of Pregnancy Act -
I. Marriage proposal is made by or on behalf 1971
of the concerned parties. • The Pre-Conception and Pre- Natal
II. Acceptance of the proposal in the presence Diagnostic Techniques (Prohibition of sex
of one or two male and two female selection (PCPNDT)) Act, 1994 &2002 -
witnesses. 1994
III. Settlement of Dowry or Mehr. • Indian Penal Code - 1860
Choose the correct option • Amended Immoral Traffic (Prevention) Act
1) I only 2) I and II only - 1986
3) II and III only 4) All the above • Indecent Representation of Women
Answer: 4
(Prohibition) Act - 1986
• The Commission of Sati (Prevention) Act -
Explanation: 1987
• The essential elements of Muslim marriage • The Protection of Women from Domestic
are Violence Act - 2005
• Marriage proposal is made by or on behalf • The Prohibition of Child Marriage Act -
of the concerned parties. Hence, 2006
statement I is correct. • Sexual Harassment (Prevention,
• Acceptance of the proposal in the presence Prohibition and Redressal) of Women at
of one or two male and two female Workplace Act - 2019
witnesses. Hence, statement II is correct. 74. Which of the following pair is incorrect
• Settlement of Dowry or Mehr. Hence, regarding the river tribunals and states
statement III is correct. covered?
• Marriage among Muslims is contract. It is 1) Ravi & Beas – Punjab, Haryana and
an obligation to be fulfilled. Rajasthan

AMIGOS IAS 28
2) Kaveri – Karnataka, Kerala, Tamil Nadu and • In 1976, the Untouchability (Offences) Act,
Puducherry 1955 has been comprehensively amended
3) Vamshadara – Chhattisgarh and Andhra and renamed as the Protection of Civil
Pradesh Rights Act, 1955 to enlarge the scope and
4) Mahanadi – Chhattisgarh and Orissa make penal provisions more stringent. The
Answer: 3 act defines civil right as any right accruing
to a person by reason of the abolition of
Explanation:
untouchability by Article 17 of the
• Krishna River Waters Tribunal – 1969:
Constitution. Hence, statement I is correct.
Maharashtra, Karnataka, Andhra Pradesh
• The term ‘untouchability’ has not been
Telangana
defined either in the Constitution or in the
• Godavari River Waters Tribunal – 1969:
Act. However, the Mysore High Court held
Maharashtra, Karnataka, Andhra Pradesh
that the subject matter of Article 17 is not
Madhya Pradesh, Orissa, Telangana
untouchability in its literal or grammatical
• Narmada Waters Tribunal – 1969: sense but the ‘practice as it had developed
Rajasthan, Gujarat, Madhya Pradesh & historically in the country’. It refers to the
Maharashtra social disabilities imposed on certain
• Ravi & Beas – 1986: Punjab, Haryana & classes of persons by reason of their birth
Rajasthan in certain castes. Hence, it does not cover
• Kaveri – 1990: Karnataka, Kerala, Tamil social boycott of a few individuals or their
Nadu, Puducherry exclusion from religious services, etc.
• Vamshadara – 2010: Orissa, Andhra • The Supreme Court held that the right
Pradesh under Article 17 is available against private
• Mandavi-2010: Goa, Karnataka, individuals and it is the constitutional
Maharashtra obligation of the State to take necessary
• 2nd Krishna Tribunal-2004: Maharashtra, action to ensure that this right is not
Karnataka, Andhra Pradesh, Telangana violated. Hence, statement II is incorrect.
• Mahanadi-2018: Chhattisgarh and Orissa 76. Which of the following statement is correct
75. Consider the following statements regarding Article 24?
I. The Untouchability (Offences) Act, 1955 1) It prohibits employment of children below
has been comprehensively amended and 14 years in all factories.
renamed as the Protection of Civil Rights 2) It prohibits employment of children below
Act, 1955 in 1976. 18 years in all factories.
II. The right under Article 17 is not available 3) It prohibits employment of children below
against private individuals. 18 years in hazardous activities.
Which of the above given statements is/are 4) It prohibits employment of children below
correct? 14 years in any factory, mine or hazardous
1) I only 2) II only activities.
3) Both I and II 4) Neither I nor II Answer: 4
Answer: 1 Explanation:
Explanation: • Prohibition of Employment of Children in
Abolition of Untouchability Factories, etc. Article 24 prohibits the
employment of children below the age of
• Article 17 abolishes ‘untouchability’ and
14 years in any factory, mine or other
forbids its practice in any form. The
hazardous activities like construction work
enforcement of any disability arising out of
or railway. But it does not prohibit their
untouchability shall be an offence
employment in any harmless or innocent
punishable in accordance with law.
work.

AMIGOS IAS 29
• The Child Labour (Prohibition and Explanation:
Regulation) Act, 1986, is the most • Article 244 in Part X of the Constitution
important law in this direction. In addition, envisages a special system of
the Employment of Children Act, 1938; the administration for certain areas designated
Factories Act, 1948; the Mines Act, 1952; as ‘scheduled areas’ and ‘tribal areas.
the Merchant Shipping Act, 1958; the • The Fifth Schedule of the Constitution
Plantation Labour Act, 1951; the Motor deals with the administration and control
Transport Workers Act, 1951; Apprentices of scheduled areas and scheduled tribes in
Act, 1961; the Bidi and Cigar Workers Act, any state except the four states of Assam,
1966; and other similar acts prohibit the Meghalaya, Tripura and Mizoram.
employment of children below certain age. • The Sixth Schedule of the Constitution, on
• In 1996, the Supreme Court directed the the other hand, deals with the
establishment of Child Labour administration of the tribal areas in the
Rehabilitation Welfare Fund in which the four northeastern states of Assam,
offending employer should deposit a fine Meghalaya, Tripura and Mizoram.
of ₹20,000 for each child employed by him. 78. Consider the following statements regarding
It also issued directions for the the Pradhan Mantri Jan Vikas Karyakram
improvement of education, health and
I. It is a Centrally Sponsored Scheme, which
nutrition of children.
is being implemented by the Ministry of
• The Commissions for Protection of Child Minority Affairs.
Rights Act, 2005 was enacted to provide for
II. Its implementation period is FY 2022-23 to
the establishment of a National
2025-26.
Commission and State Commissions for
III. Projects under this scheme are approved in
Protection of Child Rights and Children’s
the identified areas where the
Courts for providing speedy trial of
concentration of minority population is
offences against children or of violation of
more than 50% in the catchment area.
child rights.
Which of the above given statements is/are
• In 2006, the government banned the
correct?
employment of children as domestic
servants or workers in business 1) I only 2) I and II only
establishments like hotels, dhabas, 3) II and III only 4) All the above
restaurants, shops, factories, resorts, spas, Answer: 2
tea-shops and so on. It warned that anyone Explanation:
employing children below 14 years of age Pradhan Mantri Jan Vikas Karyakram
would be liable for prosecution and penal • Pradhan Mantri Jan Vikas Karyakram
action. (PMJVK) is a Centrally Sponsored Scheme,
• The Child Labour (Prohibition and which is being implemented by the
Regulation) Amendment Act, 2016, Ministry of Minority Affairs with the
amended the Child Labour (Prohibition and objective to develop infrastructure
Regulation) Act, 1986. It has renamed the projects, which are community assets, in
Principal Act as the Child and Adolescent the identified areas with development
Labour (Prohibition and Regulation) Act, deficits for socio-economic development
1986. of the said areas. Hence, statement I is
77. Which of the following state does not come correct.
under Sixth Schedule? • In 2022-23, the revised PMJVK has been
1) Meghalaya 2) Tripura approved by the Government for
3) Mizoram 4) Manipur continuation over the 15th Finance
Answer: 4 Commission Cycle i.e., during FY 2022-23 to
2025-26. Hence, statement II is correct.

AMIGOS IAS 30
• The Revised PMJVK Scheme is Muslim Women (Protection of Rights on
implemented for all Districts of the Country Marriage) Act, 2019.
including all the Aspirational Districts. 80. As per 2011 Census, the child labour is
• Projects are approved in the identified highest in which state?
areas where the concentration of minority 1) Bihar
population is more than 25% in the 2) Rajasthan
catchment area (15 KM radius). Hence, 3) Maharashtra
statement III is incorrect.
4) Uttar Pradesh
• In 2022-23, projects approved under the
Answer: 4
scheme includes school buildings,
Explanation:
residential schools, hostels, ITIs, skill
centres, health projects including • Together, Uttar Pradesh, Bihar, Rajasthan,
hospitals, health centres, Sadbhav Maharashtra, and Madhya Pradesh
Mandap, Community Hall, sports projects constitute nearly 55% of total working
like sports complex, working women children in India.
hostels, etc. Ministry of Minority Affairs in 81. Consider the following statements regarding
collaboration with National Remote the Sexual Harassment of Women at
Sensing Centre, ISRO has started geo Workplace (Prevention, Prohibition and
tagging of the infrastructure constructed Redressal) Act, 2013.
under the scheme. I. Vishakha guidelines formed the basis for
79. Consider the following regarding divorce this Act.
among Muslims II. Employers are required to constitute an
I. Khula Internal Complaints Committee at each
II. Mubarat workplace with 10 or more employees.
In which of the above types of divorce, Which of the above given statements is/are
initiation starts from woman side? correct?
1) I only 2) II only 1) I only 2) II only
3) Both I and II 4) Neither I nor II 3) Both I and II 4) Neither I nor II
Answer: 1 Answer: 3
Explanation: Explanation:
Divorce system among Muslims: Background:
• As per Muslim law but without the • The Supreme Court in a landmark
interference of the court judgment in the Vishakha and others v
State of Rajasthan 1997 case gave Vishakha
• Divorce initiation from woman is called as
Guidelines. These guidelines formed the
Khula. If it is either from both then it is
basis for the Sexual Harassment of Women
called as Mubarat.
at Workplace (Prevention, Prohibition and
• Talaq: Can only be initiate and use by
Redressal) Act, 2013. Hence, statement I is
husband.
correct.
• Talaq-e-hasan: Uttering the word talaq by
• The SC also drew its strength from several
husband followed by period of conjugal
provisions of the Constitution including
abstinence till the completion of iddat.
Article 15 (against discrimination on
• Talaq-e-ahasan: Three pronouncements grounds only of religion, race, caste, sex,
made during three successive period of and place of birth), also drawing from
menstruation and no sexual intercourse relevant International Conventions and
during this time. norms such as the General
• Note: Talaq-e-ul-Bidat is the third type Recommendations of the Convention on
which was recently abolished by Supreme the Elimination of All Forms of
Court and later government through The

AMIGOS IAS 31
Discrimination Against Women (CEDAW), component of Nari Adalats - women's
which India ratified in 1993. collectives to promote and facilitate
Key Provisions: alternative dispute resolution and gender
• Prevention and Prohibition: The Act places justice in society and within families.
a legal obligation on employers to prevent Hence, statement I and II are correct.
and prohibit sexual harassment in the 83. Which of the following districts in Andhra
workplace. Pradesh are officially declared as tribal
• Internal Complaints Committee (ICC): districts?
Employers are required to constitute an I. Parvatipuram Manyam
ICC at each workplace with 10 or more II. Srikakulam
employees to receive and address III. Vizianagaram
complaints of sexual harassment. Hence, IV. Alluri Sitharama Raju
statement II is correct. Choose the correct option
• The Complaints Committees have the 1) I and II 2) II and III
powers of civil courts for gathering
3) III and IV 4) I and IV
evidence.
Answer: 4
• Duties of Employers: Employers must
Explanation:
undertake awareness programs, provide a
safe working environment, and display • The state government is contemplating
information about the POSH Act at the granting another district for tribal areas
workplace. with Rampa Chodavaram as headquarters.
• Complaint Mechanism: The Act lays down • The government has already carved out
a procedure for filing complaints, two exclusive districts for tribal agency
conducting inquiries, and providing a fair areas-Alluri Sitharama Raju district and
opportunity to the parties involved. Parvatipuram Manyam district covering
the entire tribal villages from erstwhile
• Penalties: Non-compliance with the Act's
Visakhapatnam, Vizianagaram and East
provisions can result in penalties, including
Godavari districts.
fines and cancellation of business licenses.
84. Which of the following diseases are covered
82. Consider the following statements
under National Trust Act?
I. Sambal sub-scheme is for safety and
I. Autism
security of women.
II. Leprosy
II. Samarthya sub-scheme is for
empowerment of women. III. Cerebral Palsy
Which of the above given statements is/are IV. Mental Retardation
incorrect? Choose the correct option
1) I only 2) II only 1) I, II and III only 2) I, III and IV only
(3) Both I and II 4) Neither I nor II 3) II, III and IV only 4) All the above
Answer: 4 Answer: 2
Explanation: Explanation:
• ‘Mission Shakti’ has two sub-schemes - National Trust Act
'Sambal' and 'Samarthya'. While the • The National Trust is a statutory body of
"Sambal" sub-scheme is for safety and the Ministry of Social Justice and
security of women, the "Samarthya" sub- Empowerment, Government of India, set
scheme is for empowerment of women. up under the “National Trust for the
The components of 'Sambal' sub-scheme Welfare of Persons with Autism, Cerebral
consist of erstwhile schemes of One Stop Palsy, Mental Retardation and Multiple
Centre (OSC), Women Helpline (WHL), Beti Disabilities” Act (Act 44 of 1999).
Bachao Beti Padhao (BBBP) with a new Vision of The National Trust

AMIGOS IAS 32
• An inclusive society which values human • To evolve procedures for appointments of
diversity and enables and empowers full guardians and trustees for persons with
participation of Persons with Disability to disabilities.
live independently with dignity, equal 85. The book “Social Change in Modern India”
rights and opportunities. was written by?
• The National Trust’s vision statement 1) S.C Dube 2) M.N Srinivas
reflects a very changed India for all Indians 3) G.S Ghurye 4) D.N Majumdar
and is based on a human right, i.e. Answer: 2
UNCRPD, approach. The vision statement
Explanation:
focuses NT as being a leader in the
M.N. Srinivas
disability sector in India. As laws focusing
on Persons with Disability are changing, • “Religion and Society among the Coorgs of
focused leadership needs to be South India”
established. Working through local and • Social change in Modern India
regional registered organizations and local • Caste in Modern India and other Essays
level committees, the National Trust must 86. Consider the following statements regarding
make its vision and focus clear throughout the Atal Vayo Abhyuday Yojana
the country. I. Under the Integrated Programme for
Mission of the National Trust Senior Citizens scheme, assisted living
• The National Trust works towards devices are distributed free of cost, in
providing opportunities for capacity camp mode, to senior citizens.
development of Persons with Disability II. Under the Rashtriya Vayoshri Yojana
and their families, fulfilling their rights, scheme, Grant-in-Aid is given to
facilitating and promoting the creation of Implementing Agencies for running and
an enabling environment and an inclusive maintenance of Senior Citizens Homes.
society. Which of the above given statements is/are
• The National Trust’s mission, or correct?
fundamental purpose, is to create an 1) I only 2) II only
enabling environment, i.e. providing 3) Both I and II 4) Neither I nor II
opportunities for Persons with Disabilities Answer:4
through comprehensive support systems
Explanation:
which can also be done by collaborating
• The Umbrella Scheme of Atal Vayo
with other Ministries, etc., which will lead
Abhyuday Yojana (AVYAY), being run by the
towards development of an inclusive
Department of Social Justice and
society.
Empowerment, includes components to
Objectives
provide financial security, healthcare,
• The objectives of the National Trust in nutrition, shelter, welfare etc. for senior
particular are: citizens.
• To enable and empower persons with • Under one of such components, namely,
disability to live as independently and as the Integrated Programme for Senior
fully as possible within and as close to Citizens (IPSrC), Grant-in-Aid is given to
their community as possible; Implementing Agencies for running and
• To facilitate the realisation of equal maintenance of Senior Citizens Homes
opportunities, protection of rights and full where basic amenities like shelter, food,
participation of persons with disability; medical care, entertainment opportunities
• To extend support to its registered etc. are provided free of cost to indigent
organisations to provide need-based senior citizens. Hence, statement I is
services; and incorrect.

AMIGOS IAS 33
• Under Rashtriya Vayoshri Yojana (RVY), • To take steps to secure the participation of
assisted living devices are distributed free workers in the management of industries
of cost, in camp mode, to senior citizens (Article 43 A).
belonging to the families living below the 88. Which of the following schemes merged in
poverty line or having Rs. 15000/- as the Pradhan Mantri Anusuchit Jaati
monthly income and suffering from age Abhuyday Yojana?
related disabilities. Hence, statement II is I. Pradhan Mantri Adarsh Gram Yojana
incorrect. II. Special Central Assistance to Scheduled
87. Which of the following articles of the Indian Castes Sub Plan
Constitution are related to the women? III. Babu Jagjivan Ram Chhatrawas Yojana
I. Article 39(d) Choose the correct option
II. Article 42 1) I only 2) I and II only
III. Article 43(A) 3) II and III only 4) All the above
Choose the correct option Answer: 4
1) I only 2) I and II only Explanation:
3) II and III only 4) All the above • Pradhan Mantri Anusuchit Jaati Abhuyday
Answer: 2 Yojana (PM- AJAY) is a merged scheme of
Explanation: 03 Centrally Sponsored Scheme namely
• To promote the welfare of the people by Pradhan Mantri Adarsh Gram Yojana
securing a social order permeated by (PMAGY), Special Central Assistance to
justice–social, economic and political and Scheduled Castes Sub Plan ( SCA to SCSP)
to minimise inequalities in income, status, and Babu Jagjivan Ram Chhatrawas
facilities and opportunities (Article 38). Yojana(BJRCY) and has been implemented
• To secure (a) the right to adequate means since 2021-22 with an aim to reduce
of livelihood for all citizens; (b) the poverty of the SC communities by
equitable distribution of material generation of additional employment
resources of the community for the opportunities through Skill development,
common good; (c) prevention of income generating schemes and other
concentration of wealth and means of initiatives and to improve socio-economic
production; (d) equal pay for equal work developmental indicators by ensuring
for men and women; (e) preservation of adequate infrastructure and requisite
the health and strength of workers and services in the SC dominated villages.
children against forcible abuse; and (f) 89. Who is the Chairman of National Foundation
opportunities for healthy development of for Communal Harmony?
children (Article 39). 1) Prime Minister
• To promote equal justice and to provide 2) Central Home Minister
free legal aid to the poor (Article 39 A). 3) President
• To secure the right to work, to education 4) Central Minority Affairs Minister
and to public assistance in cases of Answer: 2
unemployment, old age, sickness and
Explanation:
disablement (Article 41).
National Foundation for Communal Harmony
• To make provision for just and humane
• The National Foundation for Communal
conditions of work and maternity relief
Harmony, a voluntary organization with
(Article 42).
the slogans of Vasudaika Sambhava &
• To secure a living wage, a decent standard
Sarvadharma Sambhava, was set up by the
of life and social and cultural opportunities
Central Government to prevent communal
for all workers (Article 43).
conflicts. This organization provides

AMIGOS IAS 34
scholarships and arranges educational precipitation changes and increasing
rehabilitation for victims of religious extreme climate events affecting the large
persecution. And this institution observes human populations as well as endemic
Kwami Ekta Week from 19th November to biodiversity of Lake Victoria Basin.
25th November every year. The Union • Lake Victoria is the largest freshwater lake
Home Minister is the Chairman of this in Africa, bordered by Kenya, Uganda, and
organization. Tanzania, and is the primary source of the
90. What is the aim of the ‘CARINGS’ portal Nile River. Hence, Statements I, II and III are
1) To take care of women in distress correct.
2) For facilitating adoption of orphaned and
abandoned senior citizens
3) For facilitating adoption of orphaned,
abandoned and surrendered children
4) None of the above
Answer: 3
Explanation:
• The Government has put in place a
centralized online system called the Child
Adoption Resource Information and
Guidance System (CARINGS) for facilitating
adoption of orphaned, abandoned and
92. Consider the following statements about
surrendered children under the Juvenile
Financial Action Task Force (FATF):
Justice (Care and Protection of Children)
Act (JJ Act), 2015. I. It is the global money laundering and
terrorist financing watchdog set up in 1989
• Adoptions in the country are undertaken
at a G-20 meeting of developed nations in
through the Juvenile Justice (Care and
Paris.
Protection of Children) Act (JJ Act), 2015
and the Hindu Adoptions and Maintenance II. It has its headquarters in Brussels.
Act, 1956. The Ministry of Women and Which of the statements given above is/are
Child Development administers adoptions correct?
of orphaned, abandoned and surrendered 1) Only I 2) Only II
children under the JJ Act, 2015. 3) Both I and II 4) Neither I nor II
91. Consider the following statements about Answer: 4
Lake Victoria: Explanation:
I. It is the largest freshwater lake in Africa by • FATF is an inter-governmental policy-
surface area. making and standard-setting body
II. Lake Victoria is bordered by three dedicated to combating money laundering
countries: Kenya, Uganda, and Tanzania. and terrorist financing.
III. It is the primary source of the Nile River. • It was established in 1989 during the G7
Which of the statements given above are Summit in Paris to develop policies against
correct? money laundering. In 2001, its mandate
1) Only I 2) Only II expanded to include terrorism financing.
3) Only II and III 4) All I, II and III Hence, Statement I is not correct.
Answer: 4 • Headquarters: Paris, France. Hence,
Statement II is not correct.
Explanation:
• FATF members include 39 countries,
• Recently, a new scientific report published
including the United States, India, China,
in the journal Nature shows that significant

AMIGOS IAS 35
Saudi Arabia, Britain, Germany, France, • The Rapid Support Forces are paramilitary
and the EU as such. forces operated by the Government of
• India became a member of FATF in 2010. Sudan.
• FATF, as part of its efforts, maintains two • Although formally established in 2013, the
types of lists - the Greylist and the blacklist. RSF can trace its roots to the Janjaweed
• Countries known as Non-Cooperative militias, which is a group of Arab tribes
Countries or Territories (NCCTs) are put on primarily based in western Sudan.
the blacklist. These countries support • They were first armed and organised in the
terror funding and money laundering early 1980s to help the government
activities. The FATF revises the blacklist expand its influence in neighbouring civil-
regularly, adding or deleting entries. war-torn Chad.
• Countries that are considered a safe haven 94. “Persona non grata," often appearing in the
for supporting terror funding and money news, means?
laundering are put on the FATF grey list. 1) A diplomat or foreign individual whose
This inclusion serves as a warning to the entering or remaining in a certain country
country that it may enter the blacklist. has been welcomed by the Head of the
Three countries, North Korea, Iran, and State.
Myanmar are currently in FATF’s blacklist. 2) A diplomat or foreign individual whose
• No financial aid is given to them by the entering or remaining in a certain country
International Monetary Fund (IMF), the has been prohibited by that country.
World Bank, the Asian Development Bank 3) A diplomat or foreign individual whose
(ADB) and the European Union (EU). entering or remaining in a certain country
93. ‘Rapid Security Forces’, recently seen in has been non-objectionable by that
news, is related to which amongst the country.
following countries? 4) A diplomat or foreign individual whose
1) Palestine 2) Israel entering or remaining in a certain country
3) Sudan 4) Syria is immunized from punishment by that
Answer:3 country.
Explanation: Answer: 2
• Around 530 Indians have been evacuated Explanation:
from Sudan so far with 250 people taken to • The Canadian government declared a
Jeddah in Saudi Arabia via an Indian Air Chinese diplomat "persona non grata" on
Force (IAF) aircraft and 278 on Indian Navy May 8th, 2023, due to allegations of
Ship Sumedha, according to the Ministry of intimidation of a Canadian lawmaker who
External Affairs. criticised China for its human rights abuses.
• Sudan’s military and its paramilitary forces As a reciprocal countermeasure, China
have been involved in a fierce battle declared a Canadian diplomat in Shanghai
against each other since April 15, leaving at as "persona non grata."
least 420 people dead and more than 3,700 • Persona non grata in foreign diplomacy
wounded. The clashes follow months of refers to a diplomat or foreign individual
heightened tension between the Army whose entering or remaining in a certain
Chief Lt Gen Abdel Fattah al-Burhan and country has been prohibited by that
the Rapid Support Force (RSF) head Gen country.
Mohamed Hamdan Dagalo, also known as • According to Article 9 of the Vienna
Hemetti. Both the generals are at Convention for Diplomatic Relations, 1961,
loggerheads as they fight for control of “the host country can declare any member
Sudan’s major institutions. of the diplomatic staff of a mission persona

AMIGOS IAS 36
non grata at any point in time and for any was down about 15% from the previous
reason”. years.
95. The Indo-Pacific Maritime Domain • This year’s report focuses on adaptation
Awareness (IPMDA) Initiative was introduced finance, or the availability of money to
by carry out the adaptation projects.
1) ASEAN 97. "Bab-El-Mandab" is often a scene in the news
2) QUAD related to?
3) Sanghai Cooperation Organisation 1) Mediterranean Sea 2) Red Sea
4) Comprehensive and Progressive 3) Black Sea 4) South China Sea
Agreement for Trans-Pacific Partnership Answer: 2
Answer: 2 Explanation:
Explanation: • Prime Minister Narendra Modi and Israeli
• Recently, the Chief of the Naval Staff Prime Minister Benjamin Netanyahu held a
Admiral addressed the fourth edition of the telephone discussion on the escalating
Goa Maritime Conclave (GMC) where he threats to maritime security in the Red Sea
stressed that building networks and because of the actions of Houthi militants
partnerships like the Indo-Pacific Maritime of Yemen.
Domain Awareness (IPMDA) would be • The conversation came soon after the
instrumental in ensuring the security and Pentagon announced an international
stability of the Indian Ocean Region (IOR). mission to counter the growing number of
• The IPMDA, introduced by the Quad group attacks against Israel-bound international
(consisting of India, Australia, Japan, and traffic.
the US) at the Tokyo summit, 2022, aims to • It is a strait that connects the Red Sea
monitor "dark shipping" and create a more (northwest) with the Gulf of Aden and the
comprehensive and precise real-time Indian Ocean (southeast).
maritime overview of partner nations' • It is located between Arabia (northeast)
waters. and Africa (southwest).
• It focuses on integrating the Pacific Islands, • It is an important strategic connection in
Southeast Asia, and the Indian Ocean the marine commerce route connecting
region (IOR) in the Indo-Pacific. the Mediterranean Sea to the Indian Ocean
96. Who among the following releases the via the Red Sea and the Suez Canal.
Adaptation Gap Report? • It is one of the world's most important
1) UN Environment Programme seaborne commodity shipping routes,
2) UN Development Programme primarily for crude oil and petroleum.
3) International Energy Agency • Yemen borders it on the Arabian Peninsula,
4) International Renewable Energy Agency and Djibouti and Eritrea border it on the
Answer: 1 African coast.
Explanation:
• As per the latest edition of Adaptation Gap
Report, 2023 released by UN Environment
Programme, developing countries,
together, need at least USD 215 billion
every year this decade to carry out
meaningful adaptation work. In 2021, just
about USD 21 billion went to developing
countries for adaptation projects, which

AMIGOS IAS 37
98. Consider the following statements: 100. With which of the following countries has
I. The New Development Bank has been set India signed all the four foundational
up under Shanghai Cooperation agreements?
Organization (SCO). 1) Russia 2) USA
II. The headquarters of New Development 3) France 4) Australia
Bank is in Shanghai. Answer: 2
Which of the statements given above is/are Explanation:
correct? • Recently, the Prime Minister (PM) of India
1) I only 2) II only has stated that, despite occasional issues,
3) Both I and II 4) Neither I nor II India and the US have been on a positive
Answer: 2 trajectory in relations.
Explanation: • India has now signed all four foundational
• The New Development Bank (NDB) is a agreements with the USA.
multilateral financial institution • The Logistics Exchange Memorandum of
established by the BRICS countries, Agreement (LEMOA) in 2016,
namely, Brazil, Russia, India, China and • The Communications Compatibility and
South Africa. Hence, Statement I is not Security Agreement (COMCASA) in 2018
correct. • The Basic Exchange and Cooperation
• NDB is headquartered in Shanghai, China. Agreement for Geo-Spatial cooperation
Hence, Statement II is correct. (BECA) in 2020.
99. “Free Movement Regime," often seen in the • While the General Security of Military
news, is related to the affairs of India and ? Information Agreement (GSOMIA) was
1) USA 2) Myanmar signed a long time ago, an extension to it,
3) European Union 4) Pakistan the Industrial Security Annex (ISA), was
Answer: 2 signed in 2019.
Explanation: 101. Which of the following schemes has been
launched recently by the Prime Minister for
• India’s recent plans to review the Free
the protection of Ramsar sites?
Movement Regime (FMR) agreement with
Myanmar and fence the India-Myanmar 1) Amrit Bharat Yojana
border have initiated discussions, 2) Amrit Sarovar Scheme
especially in the northeastern states. 3) Amrit Dharohar Yojana
• Currently, India and Myanmar share a 4) Abhinav Ramsar Yojana
1,643 km border across Manipur, Mizoram, Answer: 3
Nagaland, and Arunachal Pradesh, of which Explanation:
only 10 km is fenced in Manipur. • Amrit Dharohar Yojana scheme was
• The FMR was established in 2018 as part of announced in the 2023–24 budget for a
India's Act East policy, promoting cross- period of three years. The scheme,
border movement up to 16 km without a identifying the importance of the wetlands,
visa. aims to promote their optimal use. Under
• Individuals residing at the border need a this scheme, the communities will be at the
one-year border pass for stays lasting up to centre of wetland protection.
two weeks in the neighbouring country. • This scheme will focus on enhancing the
• It aimed to facilitate local border trade, biodiversity in the wetlands, preserving
improve access to education and and increasing their carbon stock, building
healthcare for border residents, and upon their eco-tourism potential, and
strengthen diplomatic ties. enabling the local communities to

AMIGOS IAS 38
optimally use the wetlands for II. Pawan Kumar : Kabaddi
employment generation. III. Diksha Dagar : Golf
102. For which of the following short stories has IV. Sunil Kumar : Wrestling
Patanjali Sastri won the 2023 Sahitya Which of the above pairs is/are correct?
Academi Award? 1) Only I and II 2) Only II and III
1) Tana Margam 3) Only I and III 4) All the above
2) Gaalivana Answer: 4
3) Astitvanadam Aavali Teerana Explanation:
4) Rameshwaram Kaakulu Marikonni Kathalu • Nasreen Shaikh, a standout athlete in Kho-
Answer: 4 Kho
Explanation: • Pawan Kumar belongs to kabaddi.
• The 2023 Sahitya Akademi Award for • Diksha Dagar belongs to Golf.
Telugu was given to writer Patanjali Sastry • Sunil Kumar belongs to Wrestling. Hence
for his short story “Rameshwaram Kaakulu all four pairs are correct.
Marikonni Kathalu."
105. The PRANA portal has been continuously in
• The award is sponsored by the Sahitya the news in recent times. Which of the
Akademi, a government of India following statements best explains its
institution. The award includes a cash prize objective?
of ₹1 lakh (US$1,300), a copper plaque, and
1) It is a portal for monitoring of
a shawl.
implementation of National Clean Air
• The award aims to recognise and promote Programme.
excellence in Indian writing.
2) It will provide online education to far flung
103. Consider the following statements: areas where physical infrastructure of
I. Lucknow is the India’s first Artificial education is not present.
intelligence city. 3) It will provide the Gram Panchayats with a
II. India has got its first ever AI school in single interface to prepare and implement
Kerala. their Gram Panchayat Development Plan.
Which of the above statement is/are correct? 4) It will ensure hassle-free disbursement of
1) I only 2) II only pension to retired state government
3) Both I and II 4) Neither I nor II employees.
Answer: 3 Answer: 1
Explanation: Explanation:
• India has its first ever AI school in Kerala. • PRANA (Portal for Regulation of Air-
This pioneering development was pollution in non-attainment cities) is a
launched by Santhigiri Vidhyabhavan in portal for monitoring of the
Kerala's capital city, Thiruvananthapuram. implementation of the National Clean Air
Hence, statement II is correct. Programme (NCAP). Hence, option 1 is the
• Uttar Pradesh's government has approved correct answer.
the development of Lucknow as India's first • It will support tracking of physical as well as
artificial intelligence (AI) city. The goal is to financial status of city air action plan
revolutionise urban living by establishing implementation and disseminate
Lucknow as a hub for tech ideas and a new information on air quality management
IT hub. Hence, statement I is correct. efforts under NCAP to public.
104. Consider the following statements with • It would also disseminate information on
respect to Arjuna Awardees 2023 with their air quality to the public while also providing
sports or games: comprehensive information related to
I. Nasreen :Kho-Kho NCAP programme details, progress, city

AMIGOS IAS 39
action plans, implementation updates by and empowerment of the women,
city/state/ national level agencies, air including the girls.
quality data, and trends, etc. • The minimum investment amount is Rs.
106. The Food Safety and Standards Authority of 1,000 and the maximum amount is Rs. 2
India (FSSAI) comes under which of the lakhs. The scheme will mature in two years
following ministries? from the date of opening the account.
1) Ministry of Health and Family Welfare Interest rate for MSSC is 7.5% per annum,
2) Ministry of Food Processing Industries compounded quarterly. The scheme is
3) Ministry of Commerce and Industry operational during the period 01.04.2023
to 31.03.2025.
4) It is an independent body and not part of
any ministry. 108. Which of the following is correct with respect
to the Article-1 of Indian Constitution?
Answer: 1
1) It mentions India as its only name, and the
Explanation:
President can use “Bharat” at her pleasure.
• FSSAI is under Ministry of Health & Family
2) India shall be a Federation of States.
Welfare. Hence option (1) is the correct
answer. 3) The States and the territories thereof shall
be as specified in the Second Schedule.
• It is an autonomous statutory body
established under Food Safety and 4) The territory of India shall comprise the
Standards Act (FSSA), 2006. territories of states, union territories, and
territories as may be acquired.
• FSSA 2006 consolidates various acts &
orders that had earlier handled food Answer: 4
related issues in various Ministries. Explanation:
• It regulates and monitor, manufacture, • A possible name change for India has been
processing, distribution of food while a topic of debate for some time. Some
ensuring safe and wholesome food to people argue that the name Bharat better
consumers. reflects the nation's identity and culture,
• Headquarters – Delhi while others maintain that India is a well-
established and internationally recognised
107. Recently, which of the following schemes has
name.
been launched by the central government to
make women self-reliant? • India, that is Bharat, shall be a Union of
States.
1) Dharani Matruthva yojana
• The States and the territories thereof shall
2) Mahila Samman Abhyudhaya yojana
be as specified in the First Schedule.
3) Mahila Samman Savings Certificate
• The territory of India shall comprise-(a)The
Scheme
territories of the States;(b)the Union
4) Beti Bachao Beti Padhao
territories specified in the First Schedule;
Answer: 3 and(c)such other territories as may be
Explanation: acquired.
• The Mahila Samman Saving Certificate 109. Indian Railway has set a target of becoming
(MSSC) is a new small savings scheme Net Zero Carbon Emitter by
launched by Ministry of Finance, Govt. of 1) 2025 2) 2027
India aimed at encouraging women
3) 2030 4) 2035
investors.
Answer: 3
• Central government has introduced this
Explanation:
scheme in the Budget 2023 to mark the
Azadi Ka Amrit Mahotsav and is a • Indian Railway has set a target of becoming
significant step towards financial inclusion Net Zero Carbon Emitter by 2030.

AMIGOS IAS 40
• IR has taken a number of initiatives to ● Disha App is managed by the Police
reduce its carbon emissions which include Department while Abhayam App (Project)
use of energy efficient technologies like will continue to be under the Transport
completely switching over to production of Department. . Hence, Statement I is not
three phase electric locomotives with correct.
regenerative features, use of head on Source:
generation (HOG) technology, use of LED https://www.thehansindia.com/news/cities/v
lights in buildings and coaches, star rated isakhapatnam/abhayam-app-for-women-
appliances and afforestation. on-move-785003
110. The quadrivalent Human Papilloma Virus
(qHPV) vaccine was developed against? 112. An inscription with the term “Bharatiyanam”
1) Blood cancer 2) Liver cancer has been found in which of the following
3) Cervical cancer 4) Mouth cancer places?
Answer: 3 1) Mangalagiri 2) Ghantasala
Explanation: 3) Mangapatnam 4) Narsipatnam
• India’s 1st indigenously developed Answer: 2
quadrivalent Human Papilloma Virus Explanation :
(qHPV) vaccine against cervical cancer. ● Inscription with the term 'Bharatiyanam'
• Cervical cancer develops in a woman's found at Ghantasala village in Andhra
cervix. Almost all cervical cancer cases Pradesh. Hence, Option (2) is correct.
(99%) are linked to infection with high-risk ● The Archaeological Survey of India (ASI)
human papillomaviruses (HPV), an discovered an ancient inscription in
extremely common virus transmitted Ghantasala village in Krishna on an
through sexual contact. umbrella stand (chatri), dating back to the
111. Consider the following statements with first century CE district.
respect to the “Abhayam App” launched by ● The inscription is in the Prakrit language
the Andhra Pradesh government: and Brahmi characters. It mentions the
I. It is managed by the Police Department. term "Bharatiyanam Nagila sethisa putasa
II. It helps commuters during the golden hour Nigokhadha sethisa Bharata sethisa."
after road accidents. ● Bharata, the son of Nagokhada and
Which of the given statements is/are correct ? grandson of Nagila, belonging to the
1) I only 2) II Only mercantile community of Bhartiya (India),
3) Both I and II 4) Neither I nor II was given the umbrella.
Answer : 4 ● The inscription is considered significant as
Explanation : it contains the earliest epigraphical
ABHAYAM APP : reference to "Bharatiyanam," signifying
the area of Bharat (India).
● Chief Minister YS Jagan Mohan Reddy
launched the Abhayam Project (App), ● The use of the term "Bharatiyanam' is
specially designed for the safety of children noteworthy. suggesting a community or
and women travelling alone in autos and clan during that period
taxis, on November 23, 2020. Hence, Source :
Statement II is not correct. https://www.newindianexpress.com/states/a
● Initially, tracking devices were installed in ndhra-pradesh/2023/Mar/11/inscription-
1,000 autos as a pilot project in with-term-bharatiyanam-found-at-
Visakhapatnam, and this will be ghantasala-village-in-andhra-pradesh-
implemented. 113. The first generic animal medicine centre in
Andhra Pradesh is inaugurated in which of
the following cities?

AMIGOS IAS 41
1) Guntur 2) Vijayawada category families who come from
3) Tirupati 4) Kurnool economically unsound circumstances and
Answer: 2 deserve a platform where they can become
Explanation: financially self-sufficient and support their
families in any way they can.
● Minister of Animal Husbandry Sidiri
Appalaraju said that the YSR Animal ● Under this, a total of 45,000 is provided in
Medicine Network Scheme is being made 3 years for women in the age group of 45–
available for the first time in the country 60. Hence, Statement II is not correct.
with the aim of providing quality cattle Source:
generic medicines at low prices to the https://www.google.com/url?sa=t&source=w
cattle patrons in the state. eb&rct=j&opi=89978449&url=https://ww
● As part of this, 52 animal medicine centres w.myscheme.gov.in/schemes/ysrebcn&ve
are being established across the state. d=2ahUKEwjNyunV-
Minister Appalaraju inaugurated the ZCEAxVEslYBHciTBY8QFnoECB0QAQ&usg=
country's first generic animal medicine AOvVaw38DDUWcIXt-r_BlCIARU1W
centre at the office premises of the 115. Which of the following statements is/are
Director of Animal Husbandry in correct with respect to “Didayi tribe”?
Vijayawada. Hence, Option (2) is correct. I. They are one of the PVTGs from Andhra
Source: Pradesh.
https://www.google.com/url?sa=t&source=w II. They inhabit the Seshachalam Hill ranges.
eb&rct=j&opi=89978449&url=https://m.s Select the correct answer from the options
akshi.com/amp/telugu-news/andhra- given below.
pradesh/first-generic-animal-medicine- 1) Only I 2) Only II
center-country- 3) Both I and II 4) Neither I nor II
1558343&ved=2ahUKEwjb6c_E14qEAxWc Answer: 4
tlYBHemACboQFnoECBEQAQ&usg=AOvVa
Explanation:
w0uYukqZWY0S1XmeMFfo3aG
• The vulnerable Didayi tribe, ‘wrongly’
114. Consider the following statements:
bracketed under the Poraja group, craves
I. All BC women are covered under YSR EBC identity.
Nestham.
• The Didayi and the Pengu tribes, who,
II. A total sum of Rs.45,000 is provided in 3 despite being different from the rest of the
years for women in the age group of 40-60. tribes in the Eastern Ghats, have been
Which of the above statement(s) is/are listed under the tribal common head of
correct? Poraja, hurting their very existence.
1) I only 2) II only • The Didayi inhabits the Visakha Agency
3) Both I and II 4) Neither I nor II area. They are not recognised as PVTG.
Answer: 4 Hence, both the statements are not
Explanation: correct.
● The scheme aimed at improving the • There are (34) Scheduled Tribes, out of
livelihood and financial empowerment of which (7) tribal groups are categorized as
women in the OC category. Hence, Particularly Vulnerable Tribal Groups
Statement I is not correct. (PVTGs).
● This is the first-of-its-kind scheme to • They are Kondareddi, Savara, Gadaba,
benefit poor women from upper castes. Porja, Kondh, Kondareddy, and Chenchu.
● To ensure that women become self- • Settlement: The Didayis live on the
sufficient, the state government brought Rangabayalu hill range, with a total
out EBC Nestham women from OC population of between 1,900 and 2,000.

AMIGOS IAS 42
• The Pengus, on the other hand, inhabit the 117. Who among the following was appointed as
Lakshmipuram hill range, with a total the Pay Revision Commissioner of the A.P.
population of about 2,400. 12th Pay Revision Commission?
• Most villages in both the hill ranges have 1) Dr. Manmohan Singh
no road connectivity, and the tribals are 2) Venugopal Rao
forced to either trek about 18 km to reach 3) P. Ramesh
the nearest road point or be carried on 4) Narayana Rao
‘dolis’.
Answer: 1
• The geographic seclusion also hinders
Explanation:
them from availing of many government
● The state government appointed the 12th
welfare schemes.
Pay Revision Commission.
Source:
● The government, in G.O. 68, said that the
https://www.thehindu.com/news/cities/V
PRC would be headed by retired IAS officer
isakhapatnam/vulnerable-didayi-tribe-
Manmohan Singh as Pay Revision
craves-for-identity/article67543875.ece
Commissioner. The terms and conditions
of the appointment of the commissioner
116. What was the sex ratio (per 1000 males) in would be issued separately. Hence, option
Andhra Pradesh according to the National (1) is correct.
Family Health Survey 5 (NFHS 5)?
1) 1030 2) 1045
Source:
3) 985 4) 998
https://www.google.com/url?sa=t&source=w
Answer: 2 eb&rct=j&opi=89978449&url=https://ww
Explanation: w.thehindu.com/news/national/andhra-
● Andhra Pradesh's sex ratio is 1,045 females pradesh/andhra-pradesh-constitutes-12-
to 1,000 males (NFHS-5). Hence, Option (2) pay-revision-
is correct. commission/article67072586.ece/amp/&v
● Child sex ratio (CSR): 934 ed=2ahUKEwjs6sGd3o-
● CSR in rural areas stands at 957 females per EAxX9g1YBHSG9CU4QFnoECBQQAQ&usg=
1,000 males. AOvVaw3R_hMdkcQAtMOmY4q4FqYp
● Kadapa holds the worst CSR figure, with 118. Deposits of 15 rare earth metals have been
779 females per 1,000 males. found in which of the following districts of
● In the ranking, Kerala, Maharashtra, Andhra Pradesh?
Madhya Pradesh, Rajasthan, Uttarakhand, 1) Kadapa 2) Nellore
Arunachal Pradesh, and Nagaland are also 3) Krishna 4) Anantapur
included as fast movers. Answer: 4
● Among the states and UTs ranked in the Explanation:
aspirant category are Goa, Odisha, West ● The Hyderabad-based National
Bengal, Geophysical Research Institute has found
Source: large deposits of 15 rare earth elements
https://www.google.com/url?sa=t&source=w (REE) in Andhra Pradesh's Anantapur city.
eb&rct=j&opi=89978449&url=http://rchii Hence, option (4) is correct.
ps.org/nfhs/NFHS- ● REE is a key component in many electronic
5Reports/Andhra_Pradesh.pdf&ved=2ahU devices and various industrial applications,
KEwjElYbkyYyEAxVERmcHHbTTB5IQFnoEC including medical technology, aerospace,
C4QAQ&usg=AOvVaw23O57IhlMxALHSYw and defence.
74L6sP ● The light rare earth element minerals
include lanthanum, cerium,

AMIGOS IAS 43
praseodymium, neodymium, yttrium, I. It is awarded in three categories: arts and
hafnium, tantalum, niobium, zirconium, culture, literature, and social service only.
and scotdium. II. A cash prize of Rs 5 lakh, a bronze statue of
Source: YSR, a memento and a commendation
https://www.google.com/url?sa=t&source=w letter is presented to the winners of YSR
eb&rct=j&opi=89978449&url=https://ww Lifetime Achievement Award.
w.indiatoday.in/india/story/ngri- Which of the above statements is/are correct?
scientists-find-rare-earth-elements-ap- 1) Only I 2) Only II
ananthapur-2357350-2023-04- 3) Both I and II 4) Neither I nor II
08%23:~:text%3DHyderabad%252Dbased Answer: 4
%2520National%2520Geophysical%2520R
Explanation:
esearch,medical%2520technology%252C%
• The State government has announced the
2520aerospace%2520and%2520defence.
recipients of YSR Lifetime Achievement and
&ved=2ahUKEwjplfbDiY-
Achievement Awards for the year 2023.
EAxWnSWwGHYiGCvcQFnoECBAQBQ&usg
=AOvVaw05-eRCqQseBRZLMfIjYRdU • The awards have been announced for
eminent persons in different fields for the
119. Best Rural Tourism Village of India in the
third consecutive year.
Silver Category, 2023, has been awarded to
which of the following villages? • The screening committee has
recommended the names of 27 eminent
1) Kondapalli 2) Lepakshi
persons, including 23 for the Lifetime
3) Etikoppaka (4) Draksharamam
Achievement Awards and 4 for the
Answer: 2 Achievement Awards.
Explanation: • The selection was made based on the
● Lepakshi has been recognised as the Best exceptional contribution of the individuals
Tourism Village 2023 in the Silver Category. to the 7 fields like agriculture, arts and
Hence, Option (2) is correct. culture, Telugu language and literature,
● In Lepakshi, time seems to slow down, and sports, medical and health, media and
its air resonates with the echoes of the social service. Hence, Statement I is not
past, where the artisans have breathed life correct.
into its magnificent stone, the architectural • The awards will be presented on
epitome, the Lepakshi. November 1 marking the State Formation
● Lepakshi, with its alluring depiction of its Day.
tradition and culture, captures the eye of • A cash prize of Rs 10 lakh, a bronze statue
every tourist. of YSR, a memento and a commendation
Source: letter will be presented to the winners of
https://www.google.com/url?sa=t&source=w YSR Lifetime Achievement Award. Hence,
eb&rct=j&opi=89978449&url=https://ww Statement II is not correct.
w.rural.tourism.gov.in/Lepakshi-Village- • The YSR achievement Award carries a cash
Silver.html%23:~:text%3DLepakshi%2520h prize of Rs 5 lakh, a memento and a
as%2520been%2520recognized%2520as,t commendation letter.
he%2520architectural%2520epitome%252 Source:
C%2520the%2520Lepakshi.&ved=2ahUKE https://www.thehindu.com/news/nationa
wjV2Kz4io- l/andhra-pradesh/ap-governor-presents-
EAxUqd2wGHd2cBLkQFnoECBEQBQ&usg= ysr-lifetime-achievement-and-ysr-
AOvVaw2dK-cUU32P2HpGuh7AVvPo achievement-awards-2023-in-various-
120. Consider the following statements with fields/article67484006.ece
respect to YSR Awards:
Directions for questions 121 to 124:

AMIGOS IAS 44
Answer these questions on the basis of the D X + 48 Rank – 4
information given below.
E X + 107 Rank - 1
The following bar graph gives information
F X - 65 Rank – 8
about the runs scored by eight batsmen in a
season. For each batsman, the runs scored G X – 45 Rank – 7
relative (difference with respect to) to player H X + 72 Rank – 3
B is given. The players are ranked from one to The rank of player C was 5
eight, with the player scoring the highest
122. How many players scored fewer runs than
number of runs ranked first, the next one
player H?
second and so on. It is known that player F was
1) 3 2) 4
ranked eighth and player D was ranked
immediately above player C. 3) 5 4) 6
Answer: 3
120
107 Explanation:
100
 Five players scored fewer than player H.
90
123. If the average number of runs scored by the
80 eight players is 435, then how many runs did
72
65 player C score?
58 1) 431 2) 410
60
45 3) 460 4) None of these
40 Answer : 1
25 Explanation:
20 The total runs scored by the eight players are
0 = = x + 29 = 435
0
A B C D E F G H
Therefore x = 406
Player C scored 406 + 25 = 431 runs.
121. What was the rank of player C? 124. If player G scored 373 runs, how many runs
1) 4 2) 5 did the eight players together score?
3) 6 4) 7 1) 2966 2) 3532
Answer : 2 3) 3576 3) None of these
Explanation: Answer: 3
 As it is known that player F was ranked Explanation:
eight and that player D was ranked X – 45 = 373
immediately above player C, it can be Therefore x = 418
concluded that both C and D scored more
Total runs = 8x + 232 = 8 *418 + 232 = 3576
runs than player B, while player G scored
125. Find the wrong number in the series.
fewer than player B. So also, as the
difference between the runs scored 12, 33, 55, 84, 114, 147, 183
by player A, E and H and player B is more 1) 55 2) 84
than that of player F, all the three must 3) 33 4) 147
have scored more runs than player B. Answer: 1
Therefore, If player B scored 0 ‘x’ runs, the Explanation:
runs scored by the other players are as The given logic is as follows.
follows: 12+21, 33+24, 57+27, 84+30, 114+33, 147+36, 183
A X + 90 Rank – 2 Hence, the wrong number in the series is 55.
𝑪 𝑬 𝑮 𝑲 𝑴
B X Rank – 6 126. , , , __, 𝟏𝟑𝟐, 𝟏𝟖𝟐
𝟏𝟐 𝟑𝟎 𝟓𝟔
C X + 25 Rank – 5

AMIGOS IAS 45
1) 5:11 2) 6:7
1) 2) 3) 6:13 4) 4:9
Answer: 3
Explanation:
3) 4)
Let the total monthly sales of companies A and
B be Rs. 2x and Rs. 3x and their total monthly
Answer: 4 expenditures be Rs. 3y and Rs. 4y.
Explanation: Given that A’s profit = 1/5 of sales = (2x/5)
( ) ( ) ( ) ( )
, , , , , Therefore, 2x – 3y = 1/5(2x) = 4/5(2x) = 3y = y
( ) ( ) ( ) ( ) ( )
( ) = x
( )
Profit of company B
= 3x – 4y = 3x – 4 ( x) =
Hence, is the missing term in the series Hence the ratio of the profits of the two
companies are 𝑥: = 6:13
127. If in a calendar year, there are 541 days and 130. The number of digits in the square root of a
10 days a week, then how many odd days will thirteen-digit number is
be there in that year? 1) 6 2) 7
1) One 2) Two 3) 8 4) Either (a) or (b)
3) Three 4) Four Answer: 2
Answer: 1 Explanation:
Explanation: Suppose a number x has m digits
 To find the number of odd days, we have i.e., 10m-1 ≤ x < 10m
to find the remainder of 541/10. The Therefore, 102m-2 ≤ x2 < 102m
remainder is one. Hence, there is one odd
i.e., x2 has 2m or 2m -1 digits. Conversely, if a
day
number has 2m – 1 or 2m digits, its square
128. If A’s father is B, C is the father of B and D is root has m digits. Therefore, if a number
A’s mother, then how is C related to D? has 13 digits, its square root as 7 digits.
1) Father 2) Grandfather
131. If LCM (x, y, z) = (x) (y) (z), HCF (y, z) = ?
3) Father-in-law 4) Uncle 1) 1 2) 2
Answer: 3 3) X 4) Cannot be determined
 A’s father is B and mother is D. Therefore, Answer: 1
D is B’s wife and C is the father of B. Hence,
Explanation:
C is D’s father-in-law.
If the LCM of two or more numbers equals
their product, the numbers must be co-
prime, hence the HCF of any two numbers
would be 1. In the given problem,
LCM (x,y,z) = x.y.z
Therefore HCF (y,z) = 1
132. Find the remainder when 1643276569 is
129. The total monthly sales of two companies A divided by 25.
and B are in the ratio 2 : 3 and their total 1) 9 2) 19
monthly expenditures are in the ratio 3 : 4. 3) 24 4) 4
Find the ratio of the profits of the two Answer: 2
companies given that company A’s profit is Explanation:
equal to a fifth of its sales.

AMIGOS IAS 46
The remainder when any number is divided by Therefore, 1.5x = 600
25 is the remainder when the number formed 135. Dinesh started a business with Rs. 20000 and
by the last two digits of that number (i.e., 69) after 5 months, Dhiraj joined him with Rs.
is divided by 25 which is 19 45000. Dinesh received Rs. 45250 including
133. A village has 10000 people, 68% of the people 10% of the profits as commission for
are males. The total literate population of the managing the business. What amount did
village is 6000. There are 1200 more male Dhiraj receive?
illiterates than female illiterates. What per 1) 56000 2) 47250
cent of the female population are illiterates? 3) 66000 4) 36500
1) 37.25% 2) 43.75% Answer: 2
3) 47.25% 4) 33.75% Explanation:
Answer: 2 Ratio of the shares of profits is
Explanation: (20000 x 12) : (45000 x 7) = 240 : 315 = 16 : 21.
Number of males = 6800 Let the total profit be P. As Dinesh receives
Number of females = 3200 10% of this as commission, the remaining
If x males are literates, 6000 - x females are 90% of P, is shared in the ratio of 16 : 21.
literates. Hence, Dinesh’s receipts will be 16/37th of
Number of male illiterates = 6800 - x. 90% of the total profit plus his commission.
Number of female illiterates = 3200 - (6000 - 0.1p + 16/37 (0.9p) = 45250
x) = x - 2800. P = Rs. 92500
Therefore, 6800 - x = x - 2800 + 1200 Dhiraj’s share = Total profit - payment to
x = 4200 Dinesh = 92500 - 45250 = Rs. 47250
Percentage of female illiterates = (100) 136. A batsman scored an average of 54 runs in
the first 6 tests. If the first test is not counted,
= (100) = 43.75% and the seventh is counted, then his average
score goes up to 58 runs. If his score in the
134. 1000 people voted on a resolution and 10% first test was 50 runs, find his score in the
of the votes were invalid. After some seventh test
discussion the 1,000 people voted again and 1) 62 2) 57
this time there were 20% invalid votes. The 3) 88 4) 74
opponents increased by 50% while the Answer: 4
motion was now rejected by a majority which Explanation:
is 300% more than that by which it was Total score in the first 6 tests = 6 x 54 = 324
formerly passed. How many people voted runs
against the resolution after the discussion?
Total score in the last six tests = 6 x 58 = 348
1) 480 2) 450 runs.
3) 540 4) 600 Score in the seventh test = Total score in all the
Answer: 4 7 tests - Total score in the first 6 tests
Explanation: = 348 + 50 - 324 = 398 - 324 = 74
137. Three taps A, B and C together can fill an
For Against empty tank in 4 hours. After 1 hour, C is
Case (i) (900 - x) X closed and the tank is filled in 6 more hours.
Find the time in which C alone can fill the
Case (2) (800 – 1.5x) (1.5x)
empty tank.
1) 5 2) 8
(900 – 2x) (1 + 300/100) = 3x – 800 3) 7 4) 3
Therefore, 4400 = 11x = x = 400 Answer: 2

AMIGOS IAS 47
Explanation:  Since A has a start of 150 m at the time B
Work done by A, B and C in 1 hour = 1/4th. starts at the starting point, A is already
Remaining part of the tank = 1 (1/4) = ¾ 150 m ahead and hence he has only 750 -
Time taken by (A and B) to fill this 3/4th part 150 = 600 m to cover.
of the tank = 6 hours  In the time B covers 750 meters, A would
⇒ A and B together fill the tank in 6/(3/4) 8 have covered (2/3) x 750 = 500 m
hours. Now, we know A, B and C take 4  Hence, he would be 100 m from the
hours and A and B take 8 hours. finishing point. Hence, B wins the race and
Part of the tank filled by C in hour = ¼ - 1/8 = at the time he finishes the race the
1/8 distance between him and A would be 100
So, C alone can fill the tank in 8 hours metres.

138. Wages for 80 women for 60 days are Rs. Directions for question 141 to 145:
43,200. How many men must work for 40 Go through the data set given below and solve
days to earn Rs. 57,600, if the daily wages for the questions based on it.
a man is four times that of a woman? Following pie chart shows the break-up of
1) 40 2) 35 employees working in various departments of
3) 20 4) 42 an organisation. Table provides the ratio of
men to women. It is given that Total Number
Answer: 1
of Employees = 1800
Explanation:
Daily wages of a woman =
,

= Rs. 9 Percentage Break up of
Daily wages of a man = 4 * 9 = Rs. 36 Marketing employees Accounts
, Dept. Dept.
Therefore, number of men = = = 40
18% 17%
139. A man can row 3/7th of a kilometre upstream
in 36 minutes and return in 12 minutes. Find
the speed of the water current
1) 1/3 kmph 2) 5/7 kmph IT Dept.
23% Producti
3) 2/9 kmph 4) 4/9 kmph on Dept.
Answer: 2 28%
HR Dept.
Explanation: 14%
Upstream speed = (3/7) (36/60) = (5/7) kmph
Ratio of Men to Women
Downstream speed = (3/7) (12/60) = (15/7)
kmph Department Men Women
Speed of the water current = (1/2) [(15/7) – Production 11 1
(5/7)] = (5/7) kmph HR 1 3
140. In a 750 m race, the ratio of speeds of two IT 5 4
runners A and B is 2 : 3. A has a head start of
Marketing 7 5
150 m. Who wins the race and by what
distance? Accounts 2 7
1) B wins the race by 20 meters 141. What is the number of men working in the
2) A wins the race by 120 meters Marketing department?
3) B wins the race by 100 meters 1) 132 2) 174
4) A wins the race by 110 meters 3) 126 4) 189
Answer: 3 Answer: 4
Explanation: Explanation:

AMIGOS IAS 48
Number of men working in the Marketing Number of people working in the Accounts
department department: 306
= 1800 x x = 189 Number of men working in the accounts
department: 68
Total number of employees be 1800
Hence required ratio = 68:306 = 2:9
Production = 504
145. The number of men working the Production
HR = 252
department of the organisation forms what
IT = 414 percentage of total number of employees
Marketing = 324 working in that department? (Rounded off
Accounts = 306 two digits after decimal)
142. The number of women working in the IT 1) 89.76 2) 91.67
department of the Organisation forms 3) 88.56 4) 94.29
approximately what per cent of the total Answer: 2
number of employees in the Organisation
Explanation:
from all departments together?
Required percentage = 11/12 * 100 = 91.67
1) 7 2) 10
3) 19 4) 15
Directions for questions 146 & 147:
Answer: 2
In each question below is given a statement
Explanation:
followed by two courses of action numbered I
Number of women working in IT department and II. You have to assume everything in the
= 1800 x x = 184 statement to be true. Then decide which of
the two suggested courses of action logically
Hence required percentage = x 100 = 10
follows for pursuing.
Given Answers
143. What is the respective ratio of the number of 146. Statement:
women working in the HR department of the
A lot of unscrupulous and non-affiliated
organisation and the total number of
colleges have started to lure unsuspecting
employees in the department?
students by giving attractive advertisements.
1) 3:4 2) 2:5
Courses of Action
3) 2:9 4) 3:7
I. Students should make appropriate enquiries
Answer: 1 while enrolling in any course.
Explanation: II. The government should initiate strict action
Total number of employees in the department against such college authorities.
= 252 1) If only I follows
Number of women working in the HR 2) If only II follows
department = 252 * ¾ = 189
3) If both I and II follow
Ratio of women working the HR to total
4) If neither I nor II follows
number of employees = 189 : 252
Answer: 3
Hence the required ratio = 3:4
Explanation:
144. What is the respective ratio of the number of
Both the courses of action are logical and
men working in the Accounts department to
the total number of employees working that recommended in the given situation.
Hence, option (c) is correct.
department?
1) 9:2 2) 7:6 147. Statement:
3) 2:9 4) 6:7 The U.S. and its allies have been claiming that
Iran is developing dangerous chemical,
Answer: 3
biological and nuclear weapons.

AMIGOS IAS 49
Courses of Action I. The University authorities have never
I. The international community should boycott taken such strong actions earlier.
Iran. II. The examination administration
II. The U.S. should wage war against Iran just committee had prior knowledge about this
as it did against Iraq a few years ago. issue.
1) If only I follows Answer: 4
2) If only II follows Explanation :
3) If both I and II follow Neither assumption is implied.
4) If neither I nor II follows 150. Obtain the missing term B, G, K, ?
Answer: 4 1) N 2) P
Explanation: 3) M 4) L
Neither course of action is logical because Answer: 1
both are too drastic just on the basis of Explanation:
claims made by one party against the  The series is defined by the 2nd, 7th, 11th
other. Hence, option (d) is correct. term of the alphabet. Hence, the next
Directions for questions 148 & 149: term would be the 14th term i.e., N. Option
In each question below is given a statement (a) is the correct answer.
followed by two assumptions numbered I and
II. Consider the statement and decide which of
the given assumption is implicit.
Given answer
148. Statement:
It is clear from the past records of the gym that
most people who exercise at the gym regularly
maintain the perfect correlation between
height and weight.
Assumptions
I. Exercise at the gym is essential for everyone.
II. If one wants to have weight according to his
height, he should exercise at the gym
regularly.
1) If only assumption I is implicit.
2) If only assumption II is implicit.
3) If both I and II are implicit.
4) If neither I nor II is implicit.
Answer: 4
Explanation:
There could be many different ways of
maintaining the perfect correlation
between height and weight. Neither
assumption is valid.
149. Statement:
Many students were caught red-handed and
rusticated while using unfair means during the
recent university examinations.
Assumptions

AMIGOS IAS 50

You might also like